1
829918
6TSL02
829918
The SAT
®
Practice
Test
#
1
Make time to take the practice test.
It is one of the best ways to get ready
for the
SAT.
After you have taken the practice test, score it
right away at sat.org/scoring.
This version of the SAT Practice Test is for students who will be taking
the digital SAT in nondigital format.
6TSL02
1
829918
6TSL02
Test begins on the next page.
......................................................................................................................................................................................................
Unauthorized copying or reuse of any part of this page is illegal.
CO N T I N U E
2
Module
1
Readin
g and Writing
33 QUESTIONS
DIRECTIONS
Th
e questions in this section address a number of important reading and writing skills. Each
question includes one or more passages, which may include a table or graph. Read each passage
and question carefully, and then choose the best answer to the question based on the passage(s).
All questions in this section are multiple-choice with four answer choices. Each question has a
single best answer.
1
Former astronaut Ellen Ochoa says that although she
doesn’t have a definite idea of when it might happen,
she
_______ that humans will someday need to be
able to live in other environments than those found
on Earth. This conjecture informs her interest in
future research missions to the moon.
Which choice completes the text with the most
logical and precise word or phrase?
A) demands
B) speculates
C) doubts
D) establishes
2
Beginning in the 1950s, Navajo Nation legislator
Annie Dodge Wauneka continuously worked to
promote public health; this
_______ effort involved
traveling throughout the vast Navajo homeland
and writing a medical dictionary for speakers of
Diné
bizaad
, the Navajo language.
Which choice completes the text with the most
logical and precise word or phrase?
A) impartial
B) offhand
C) persistent
D) mandatory
3
Following the principles of community-based
participatory research, tribal nations and research
institutions are equal partners in health studies
conducted on reservations. A collaboration between
the Crow Tribe and Montana State University
_______ this model: tribal citizens worked alongside
scientists to design the methodology and continue to
assist in data collection.
Which choice completes the text with the most
logical and precise word or phrase?
A) circumvents
B) eclipses
C) fabricates
D) exemplifies
4
The parasitic dodder plant increases its reproductive
success by flowering at the same time as the host
plant it has latched onto. In 2020, Jianqiang Wu and
his colleagues determined that the tiny dodder
achieves this _______ with its host by absorbing and
utilizing a protein the host produces when it is about
to flower.
Which choice completes the text with the most
logical and precise word or phrase?
A) synchronization
B) hibernation
C) prediction
D) moderation
5
Given that the conditions in binary star systems
should make planetary formation nearly impossible,
it’s not surprising that the existence of planets in
such systems has lacked _______ explanation.
Roman Rafikov and Kedron Silsbee shed light on
the subject when they used modeling to determine
a complex set of factors that could support
planets’ development.
Which choice completes the text with the most
logical and precise word or phrase?
A) a discernible
B) a straightforward
C) an inconclusive
D) an unbiased
6
Seminole/Muscogee director Sterlin Harjo
_______ television’s tendency to situate Native
characters in the distant past: this rejection is evident
in his series Reservation Dogs, which revolves around
teenagers who dress in contemporary styles and
whose dialogue is laced with current slang.
Which choice completes the text with the most
logical and precise word or phrase?
A) repudiates
B) proclaims
C) foretells
D) recants
...............................................................................................................................................................................................................................................................................................................
Unauthorized copying or reuse of any part of this page is illegal.
CO N T I
N U E
3
7
In 2007, computer scientist Luis von Ahn was
working on converting printed books into a digital
format. He found that some words were distorted
enough that digital scanners couldn’t recognize
them, but most humans could easily read them.
Based on that finding, von Ahn invented a simple
security test to keep automated “bots” out of
websites. The first version of the reCAPTCHA test
asked users to type one known word and one of the
many words scanners couldn’t recognize. Correct
answers proved the users were humans and added
data to the book-digitizing project.
Which choice best states the main purpose of the
text?
A) To discuss von Ahn’s invention of reCAPTCHA
B) To explain how digital scanners work
C) To call attention to von Ahn’s book-digitizing
project
D) To indicate how popular reCAPTCHA is
8
The following text is from Edith Wharton’s 1905
novel The House of Mirth. Lily Bart and a companion
are walking through a park.
Lily had no real intimacy with nature, but she
had a passion for the appropriate and could be
keenly sensitive to a scene which was the fitting
background of her own sensations.
The
landscape outspread below her seemed an
enlargement of her present mood, and she found
something of herself in its calmness, its breadth,
its long free reaches. On the nearer slopes the
sugar-maples wavered like pyres of light; lower
down was a massing of grey orchards, and here
and there the lingering green of an oak-grove.
Which choice best describes the function of the
underlined sentence in the text as a whole?
A) It creates a detailed image of the physical setting
of the scene.
B) It establishes that a character is experiencing an
internal conflict.
C) It makes an assertion that the next sentence then
expands on.
D) It illustrates an idea that is introduced in the
previous sentence.
...............................................................................................................................................................................................................................................................................................................
Unauthorized copying or reuse of any part of this page is illegal.
CO N T I N
U E
4
9
A study by a team including finance professor
Madhu Veeraraghavan suggests that exposure to
sunshine during the workday can lead to overly
optimistic behavior.
Using data spanning from 1994
to 2010 for a set of US companies, the team
compared over 29,000 annual earnings forecasts
to the actual earnings later reported by those
companies. The team found that the greater the
exposure to sunshine at work in the two weeks before
a manager submitted an earnings forecast, the more
the manager’s forecast exceeded what the company
actually earned that year.
Which choice best states the function of the
underlined sentence in the overall structure of the
text?
A) To summarize the results of the team’s analysis
B) To present a specific example that illustrates the
study’s findings
C) To explain part of the methodology used in the
team’s study
D) To call out a challenge the team faced in
conducting its analysis
10
The following text is adapted from Edith Nesbit’s
1906 novel The Railway Children.
Mother did not spend all her time in paying dull
[visits] to dull ladies, and sitting dully at home
waiting for dull ladies to pay [visits] to her. She
was almost always there, ready to play with the
children, and read to them, and help them to do
their home-lessons. Besides this she used to write
stories for them while they were at school, and
read them aloud after tea, and she always made
up funny pieces of poetry for their birthdays and
for other great occasions.
According to the text, what is true about Mother?
A) She wishes that more ladies would visit her.
B) Birthdays are her favorite special occasion.
C) She creates stories and poems for her children.
D) Reading to her children is her favorite activity.
11
The following text is from Maggie Pogue Johnson’s
1910 poem “Poet of Our Race.” In this poem, the
speaker is addressing Paul Laurence Dunbar, a Black
author.
Thou, with stroke of mighty pen,
Hast told of joy and mirth,
And read the hearts and souls of men
As cradled from their birth.
The language of the flowers,
Thou hast read them all,
And e’en the little brook
Responded to thy call.
Which choice best states the main purpose of the
text?
A) To praise a certain writer for being especially
perceptive regarding people and nature
B) To establish that a certain writer has read
extensively about a variety of topics
C) To call attention to a certain writer’s careful and
elaborately detailed writing process
D) To recount fond memories of an afternoon spent
in nature with a certain writer
...............................................................................................................................................................................................................................................................................................................
Unauthorized copying or reuse of any part of this page is illegal.
CO N T I N
U E
5
12
“To You” is an 1856 poem by Walt Whitman. In the
poem, Whitman suggests that readers, whom he
addresses directly, have not fully understood
themselves, writing,
_______
Which quotation from “To You” most effectively
illustrates the claim?
A) “You have not known what you are, you have
slumber’d upon yourself / all your life, / Your
eyelids have been the same as closed most of the
time.”
B) “These immense meadows, these interminable
rivers, you are immense / and interminable as
they.”
C) “I should have made my way straight to you long
ago, / I should have blabb’d nothing but you, I
should have chanted nothing / but you.”
D) “I will leave all and come and make the hymns of
you, / None has understood you, but I
understand you.”
13
Born in 1891 to a Quechua-speaking family in the
Andes Mountains of Peru, Martín Chambi is today
considered to be one of the most renowned figures of
Latin American photography. In a paper for an art
history class, a student claims that Chambi’s
photographs have considerable ethnographic
value—in his work, Chambi was able to capture
diverse elements of Peruvian society, representing his
subjects with both dignity and authenticity.
Which finding, if true, would most directly support
the student’s claim?
A) Chambi took many commissioned portraits of
wealthy Peruvians, but he also produced
hundreds of images carefully documenting the
peoples, sites, and customs of Indigenous
communities of the Andes.
B) Chambi’s photographs demonstrate a high level
of technical skill, as seen in his strategic use of
illumination to create dramatic light and shadow
contrasts.
C) During his lifetime, Chambi was known and
celebrated both within and outside his native
Peru, as his work was published in places like
Argentina, Spain, and Mexico.
D) Some of the peoples and places Chambi
photographed had long been popular subjects
for Peruvian photographers.
...............................................................................................................................................................................................................................................................................................................
Unauthorized copying or reuse of any part of this page is illegal.
CO N T I
N U E
6
14
Credited Film Output of James Young Deer, Dark Cloud,
Edwin Carewe, and Lillian St. Cyr
Individual Years active
Number of films known and
commonly credited
James Young Deer 1909–1924 33 (actor), 35 (director), 10 (writer)
Dark Cloud 1910–1920 35 (actor), 1 (writer)
Edwin Carewe 1912–1934 47 (actor), 58 (director), 20 (producer), 4 (writer)
Lillian St. Cyr (Red Wing) 1908–1921 66 (actor)
Some researchers studying Indigenous actors and filmmakers in the
United States have turned their attention to the early days of cinema,
particularly the 1910s and 1920s, when people like James Young Deer,
Dark Cloud, Edwin Carewe, and Lillian St. Cyr (known professionally
as Red Wing) were involved in one way or another with numerous
films. In fact, so many films and associated records for this era have
been lost that counts of those four figures’ output should be taken as
bare minimums rather than totals; it’s entirely possible, for example,
that _______
Which choice most effectively uses data from the table to complete the
example?
A) Dark Cloud acted in significantly fewer films than did Lillian St.
Cyr, who is credited with 66 performances.
B) Edwin Carewe’s 47 credited acting roles includes only films made
after 1934.
C) Lillian St. Cyr acted in far more than 66 films and Edwin Carewe
directed more than 58.
D) James Young Deer actually directed 33 films and acted in only 10.
Unauthorized copying or reuse of any part of this page is illegal.
CO N T I N U E
7
15
Juvenile Plants Found Growing on Bare Ground and in Patches
of Vegetation for Five Species
Species
Bare
ground
Patches of
vegetation Total
Percent found in
patches of vegetation
T. moroderi 9 13 22 59.1%
T. libanitis 83 120 203 59.1%
H. syriacim 95 106 201 52.7%
H. squamatum 218 321 539 59.6%
H. stoechas 11 12 23 52.2%
Alicia Montesinos-Navarro, Isabelle Storer, and Rocío Perez-Barrales
recently examined several plots within a diverse plant community in
southeast Spain. The researchers calculated that if individual plants
were randomly distributed on this particular landscape, only about 15%
would be with other plants in patches of vegetation. They counted the
number of juvenile plants of five species growing in patches of
vegetation and the number growing alone on bare ground and
compared those numbers to what would be expected if the plants were
randomly distributed. Based on these results, they claim that plants of
these species that grow in close proximity to other plants gain an
advantage at an early developmental stage.
Which choice best describes data from the table that support the
researchers’ claim?
A) For all five species, less than 75% of juvenile plants were growing in
patches of vegetation.
B) The species with the greatest number of juvenile plants growing in
patches of vegetation was H. stoechas.
C) For T. libanitis and T. moroderi, the percentage of juvenile plants
growing in patches of vegetation was less than what would be
expected if plants were randomly distributed.
D) For each species, the percentage of juvenile plants growing in
patches of vegetation was substantially higher than what would be
expected if plants were randomly distributed.
Unauthorized copying or reuse of any part of this page is illegal.
CO N T I N U E
8
16
In the mountains of Brazil, Barbacenia tomentosa
and Barbacenia macrantha—two plants in the
Velloziaceae family—establish themselves on soilless,
nutrient-poor patches of quartzite rock. Plant
ecologists Anna Abrahão and Patricia de Britto Costa
used microscopic analysis to determine that the roots
of B. tomentosa and B. macrantha, which grow
directly into the quartzite, have clusters of fine hairs
near the root tip; further analysis indicated that these
hairs secrete both malic and citric acids. The
researchers hypothesize that the plants depend on
dissolving underlying rock with these acids, as the
process not only creates channels for continued
growth but also releases phosphates that provide the
vital nutrient phosphorus.
Which finding, if true, would most directly support
the researchers’ hypothesis?
A) Other species in the Velloziaceae family are
found in terrains with more soil but have root
structures similar to those of B. tomentosa and
B. macrantha.
B) Though B. tomentosa and B. macrantha both
secrete citric and malic acids, each species
produces the acids in different proportions.
C) The roots of B. tomentosa and B. macrantha
carve new entry points into rocks even when
cracks in the surface are readily available.
D) B. tomentosa and B. macrantha thrive even when
transferred to the surfaces of rocks that do not
contain phosphates.
17
Herbivorous sauropod dinosaurs could grow more
than 100 feet long and weigh up to 80 tons, and some
researchers have attributed the evolution of
sauropods to such massive sizes to increased plant
production resulting from high levels of atmospheric
carbon dioxide during the Mesozoic era. However,
there is no evidence of significant spikes in carbon
dioxide levels coinciding with relevant periods in
sauropod evolution, such as when the first large
sauropods appeared, when several sauropod lineages
underwent further evolution toward gigantism, or
when sauropods reached their maximum known
sizes, suggesting that
_______
Which choice most logically completes the text?
A) fluctuations in atmospheric carbon dioxide
affected different sauropod lineages differently.
B) the evolution of larger body sizes in sauropods
did not depend on increased atmospheric carbon
dioxide.
C) atmospheric carbon dioxide was higher when the
largest known sauropods lived than it was when
the first sauropods appeared.
D) sauropods probably would not have evolved to
such immense sizes if atmospheric carbon
dioxide had been even slightly higher.
...............................................................................................................................................................................................................................................................................................................
Unauthorized copying or reuse of any part of this page is illegal.
CO N T I
N U E
9
18
In documents called judicial opinions, judges explain
the reasoning behind their legal rulings, and in those
explanations they sometimes cite and discuss
historical and contemporary philosophers. Legal
scholar and philosopher Anita L. Allen argues that
while judges are naturally inclined to mention
philosophers whose views align with their own
positions, the strongest judicial opinions consider
and rebut potential objections; discussing
philosophers whose views conflict with judges’ views
could therefore
_______
Which choice most logically completes the text?
A) allow judges to craft judicial opinions without
needing to consult philosophical works.
B) help judges improve the arguments they put
forward in their judicial opinions.
C) make judicial opinions more comprehensible to
readers without legal or philosophical training.
D) bring judicial opinions in line with views that are
broadly held among philosophers.
19
Public-awareness campaigns about the need to
reduce single-use plastics can be successful, says
researcher Kim Borg of Monash University in
Australia, when these campaigns give consumers a
choice: for example, Japan achieved a 40 percent
reduction in plastic-bag use after cashiers were
instructed to ask customers whether _______
wanted a bag.
Which choice completes the text so that it conforms
to the conventions of Standard English?
A) they
B) one
C) you
D) it
20
In ancient Greece, an Epicurean was a follower of
Epicurus, a philosopher whose beliefs revolved
around the pursuit of pleasure. Epicurus defined
pleasure as “the absence of pain in the body and of
trouble in the _______ that all life’s virtues derived
from this absence.
Which choice completes the text so that it conforms
to the conventions of Standard English?
A) soul,” positing
B) soul”: positing
C) soul”; positing
D) soul.” Positing
21
British scientists James Watson and Francis Crick
won the Nobel Prize in part for their 1953 paper
announcing the double helix structure of DNA, but it
is misleading to say that Watson and Crick
discovered the double helix. _______ findings were
based on a famous X-ray image of DNA fibers,
“Photo 51,” developed by X-ray crystallographer
Rosalind Franklin and her graduate student
Raymond Gosling.
Which choice completes the text so that it conforms
to the conventions of Standard English?
A) They’re
B) It’s
C) Their
D) Its
...............................................................................................................................................................................................................................................................................................................
Unauthorized copying or reuse of any part of this page is illegal.
CO N T I
N U E
10
22
In 1937, Chinese American screen actor Anna May
Wong, who had portrayed numerous villains and
secondary characters but never a heroine, finally got
a starring role in Paramount Pictures’ Daughter of
Shanghai, a film that
_______ “expanded the range
of possibilities for Asian images on screen.”
Which choice completes the text so that it conforms
to the conventions of Standard English?
A) critic, Stina Chyn, claims
B) critic, Stina Chyn, claims,
C) critic Stina Chyn claims
D) critic Stina Chyn, claims,
23
In 1637, the price of tulips skyrocketed in
Amsterdam, with single bulbs of rare varieties
selling for up to the equivalent of $200,000 in today’s
US dollars. Some historians _______ that this “tulip
mania” was the first historical instance of an asset
bubble, which occurs when investors drive prices to
highs not supported by actual demand.
Which choice completes the text so that it conforms
to the conventions of Standard English?
A) claiming
B) claim
C) having claimed
D) to claim
24
Researchers studying magnetosensation have
determined why some soil-dwelling roundworms in
the Southern Hemisphere move in the opposite
direction of Earth’s magnetic field when searching
for _______ in the Northern Hemisphere, the
magnetic field points down, into the ground, but in
the Southern Hemisphere, it points up, toward the
surface and away from worms’ food sources.
Which choice completes the text so that it conforms
to the conventions of Standard English?
A) food:
B) food,
C) food while
D) food
25
Scientists believe that, unlike most other species of
barnacle, turtle barnacles (Chelonibia testudinari)
can dissolve the cement-like secretions they use to
attach _______ to a sea turtle shell, enabling the
barnacles to move short distances across the shell’s
surface.
Which choice completes the text so that it conforms
to the conventions of Standard English?
A) it
B) themselves
C) them
D) itself
...............................................................................................................................................................................................................................................................................................................
Unauthorized copying or reuse of any part of this page is illegal.
CO N T I
N U E
11
26
The classic children’s board game Chutes and
Ladders is a version of an ancient Nepalese game,
Paramapada Sopanapata. In both games, players
encounter “good” or “bad” spaces while traveling
along a path; landing on one of the good spaces
_______ a player to skip ahead and arrive closer to
the end goal.
Which choice completes the text so that it conforms
to the conventions of Standard English?
A) allows
B) are allowing
C) have allowed
D) allow
27
In 1943, in the midst of World War II, mathematics
professor Grace Hopper was recruited by the
US military to help the war effort by solving complex
equations. Hopper’s subsequent career would involve
more than just _______ as a pioneering computer
programmer, Hopper would help usher in the digital
age.
Which choice completes the text so that it conforms
to the conventions of Standard English?
A) equations, though:
B) equations, though,
C) equations. Though,
D) equations though
28
In 1453, English King Henry VI became unfit to rule
after falling gravely ill. As a result, Parliament
appointed Richard, Third Duke of York, who had a
strong claim to the English throne, to rule as Lord
Protector. Upon recovering two years later, _______
forcing an angered Richard from the royal court and
precipitating a series of battles later known as the
Wars of the Roses.
Which choice completes the text so that it conforms
to the conventions of Standard English?
A) Henry resumed his reign,
B) the reign of Henry resumed,
C) Henry’s reign resumed,
D) it was Henry who resumed his reign,
29
Although novels and poems are considered distinct
literary forms, many authors have created hybrid
works that incorporate elements of both. Bernardine
Evaristo’s The Emperor's Babe, _______ is a verse
novel, a book-length narrative complete with
characters and a plot but conveyed in short, crisp
lines of poetry rather than prose.
Which choice completes the text with the most
logical transition?
A) by contrast,
B) consequently,
C) secondly,
D) for example,
...............................................................................................................................................................................................................................................................................................................
Unauthorized copying or reuse of any part of this page is illegal.
CO N T I
N U E
12
30
At two weeks old, the time their critical socialization
period begins, wolves can smell but cannot yet see or
hear. Domesticated dogs, _______ can see, hear, and
smell by the end of two weeks. This relative lack of
sensory input may help explain why wolves behave
so differently around humans than dogs do: from a
very young age, wolves are more wary and less
exploratory.
Which choice completes the text with the most
logical transition?
A) in other words,
B) for instance,
C) by contrast,
D) accordingly,
31
Researchers Helena Mihaljević-Brandt, Lucía
Santamaría, and Marco Tullney report that while
mathematicians may have traditionally worked
alone, evidence points to a shift in the opposite
direction. _______ mathematicians are choosing to
collaborate with their peers—a trend illustrated by a
rise in the number of mathematics publications
credited to multiple authors.
Which choice completes the text with the most
logical transition?
A) Similarly,
B) For this reason,
C) Furthermore,
D) Increasingly,
...............................................................................................................................................................................................................................................................................................................
Unauthorized copying or reuse of any part of this page is illegal.
CO N T I
N U E
13
32
While researching a topic, a student has taken the
following notes:
Pterosaurs were flying reptiles that existed
millions of years ago.
In a 2021 study, Anusuya Chinsamy-Turan
analyzed fragments of pterosaur jawbones located
in the Sahara Desert.
She was initially unsure if the bones belonged to
juvenile or adult pterosaurs.
She used advanced microscope techniques to
determine that the bones had few growth lines
relative to the bones of fully grown pterosaurs.
She concluded that the bones belonged to
juveniles.
The student wants to present the study and its
findings. Which choice most effectively uses relevant
information from the notes to accomplish this goal?
A) In 2021, Chinsamy-Turan studied pterosaur
jawbones and was initially unsure if the bones
belonged to juveniles or adults.
B) Pterosaur jawbones located in the Sahara Desert
were the focus of a 2021 study.
C) In a 2021 study, Chinsamy-Turan used advanced
microscope techniques to analyze the jawbones
of pterosaurs, flying reptiles that existed millions
of years ago.
D) In a 2021 study, Chinsamy-Turan determined
that pterosaur jawbones located in the Sahara
Desert had few growth lines relative to the bones
of fully grown pterosaurs and thus belonged to
juveniles.
33
While researching a topic, a student has taken the
following notes:
African American women played prominent roles
in the Civil Rights Movement, including at the
famous 1963 March on Washington.
Civil rights activist Anna Hedgeman, one of the
march’s organizers, was a political adviser who
had worked for President Truman.
Civil rights activist Daisy Bates was a well-known
journalist and advocate for school desegregation.
Hedgeman worked behind the scenes to make
sure a woman was included in the lineup of
speakers at the march.
Bates was the sole woman to speak, delivering a
brief but memorable address to the cheering
crowd.
The student wants to compare the two women’s
contributions to the March on Washington. Which
choice most effectively uses relevant information
from the notes to accomplish this goal?
A) Hedgeman and Bates contributed to the march
in different ways; Bates, for example, delivered a
brief but memorable address.
B) Hedgeman worked in politics and helped
organize the march, while Bates was a journalist
and school desegregation advocate.
C) Although Hedgeman worked behind the scenes
to make sure a woman speaker was included,
Bates was the sole woman to speak at the march.
D) Many African American women, including Bates
and Hedgeman, fought for civil rights, but only
one spoke at the march.
STOP
If you finish before time is called, you may check your work on this module only.
Do not turn to any other module in the test.
...............................................................................................................................................................................................................................................
Unauthorized copying or reuse of any part of this page is illegal.
14
No Test Material On This Page
......................................................................................................................................................................................................
Unauthorized copying or reuse of any part of this page is illegal.
CO N T I N U E
16
Module
2
Readin
g and Writing
33 QUESTIONS
DIRECTIONS
Th
e questions in this section address a number of important reading and writing skills. Each
question includes one or more passages, which may include a table or graph. Read each passage
and question carefully, and then choose the best answer to the question based on the passage(s).
All questions in this section are multiple-choice with four answer choices. Each question has a
single best answer.
1
For painter Jacob Lawrence, being
_______ was an
important part of the artistic process. Because he
paid close attention to all the details of his Harlem
neighborhood, Lawrence’s artwork captured nuances
in the beauty and vitality of the Black experience
during the Harlem Renaissance and the
Great Migration.
Which choice completes the text with the most
logical and precise word or phrase?
A) skeptical
B) observant
C) critical
D) confident
2
Mônica Lopes-Ferreira and others at Brazil’s
Butantan Institute are studying the freshwater
stingray species
Potamotrygon
rex to determine
whether biological characteristics such as the rays’
age and sex have _______ effect on the toxicity of
their venom—that is, to see if differences in these
traits are associated with considerable variations in
venom potency.
Which choice completes the text with the most
logical and precise word or phrase?
A) a disconcerting
B) an acceptable
C) an imperceptible
D) a substantial
3
Researchers have struggled to pinpoint specific
causes for hiccups, which happen when a
person’s diaphragm contracts
_______ . However,
neuroscientist Kimberley Whitehead has found that
these uncontrollable contractions may play an
important role in helping infants regulate their
breathing.
Which choice completes the text with the most
logical and precise word or phrase?
A) involuntarily
B) beneficially
C) strenuously
D) smoothly
4
Critics have asserted that fine art and fashion rarely
_______ in a world where artists create timeless
works for exhibition and designers periodically
produce new styles for the public to buy. Luiseño/
Shoshone-Bannock beadwork artist and designer
Jamie Okuma challenges this view: her work can be
seen in the Metropolitan Museum of Art and
purchased through her online boutique.
Which choice completes the text with the most
logical and precise word or phrase?
A) prevail
B) succumb
C)
diverge
D) intersect
5
Scholarly discussions of gender in Shakespeare’s
comedies often celebrate the rebellion of the
playwright’s characters against the rigid expectations
_______ by Elizabethan society. Most of the
comedies end in marriage, with characters returning
to their socially dictated gender roles after previously
defying them, but there are some notable exceptions.
Which choice completes the text with the most
logical and precise word or phrase?
A) interjected
B) committed
C) illustrated
D) prescribed
6
In studying the use of external stimuli to reduce the
itching sensation caused by an allergic histamine
response, Louise Ward and colleagues found that
while harmless applications of vibration or warming
can provide a temporary distraction, such _______
stimuli actually offer less relief than a stimulus that
seems less benign, like a mild electric shock.
Which choice completes the text with the most
logical and precise word or phrase?
A) deceptive
B) innocuous
C) novel
D) impractical
...............................................................................................................................................................................................................................................................................................................
Unauthorized copying or reuse of any part of this page is illegal.
CO N T I N
U E
17
7
The province of Xoconochco was situated on the
Pacific coast, hundreds of kilometers southeast of
Tenochtitlan, the capital of the Aztec Empire.
Because Xoconochco’s location within the empire
was so
_______ , cacao and other trade goods
produced there could reach the capital only after a
long overland journey.
Which choice completes the text with the most
logical and precise word or phrase?
A) unobtrusive
B) concealed
C) approximate
D) peripheral
8
The following text is from Charlotte Brontë’s 1847
novel Jane Eyre. Jane works as a governess at
Thornfield Hall.
I went on with my day’s business tranquilly; but
ever and anon vague suggestions kept wandering
across my brain of reasons why I should quit
Thornfield; and I kept involuntarily framing
advertisements and pondering conjectures about
new situations: these thoughts I did not think to
check; they might germinate and bear fruit if
they could.
Which choice best states the main purpose of the
text?
A) To convey a contrast between Jane’s outward
calmness and internal restlessness
B) To emphasize Jane’s loyalty to the people she
works for at Thornfield Hall
C) To demonstrate that Jane finds her situation
both challenging and deeply fulfilling
D) To describe Jane’s determination to secure
employment outside of Thornfield Hall
...............................................................................................................................................................................................................................................................................................................
Unauthorized copying or reuse of any part of this page is illegal.
CO N T I
N U E
18
...............................................................................................................................................................................................................................................................................................................
Unauthorized copying or reuse of any part of this page is illegal.
CO N T I N
U E
9
Text 1
Most animals can regenerate some parts of their
bodies, such as skin. But when a three-banded
panther worm is cut into three pieces, each piece
grows into a new worm. Researchers are
investigating this feat partly to learn more about
humans’ comparatively limited abilities to
regenerate, and they’re making exciting progress. An
especially promising discovery is that both humans
and panther worms have a gene for early growth
response (EGR) linked to regeneration.
Text 2
When Mansi Srivastava and her team reported that
panther worms, like humans, possess a gene for EGR,
it caused excitement. However, as the team pointed
out, the gene likely functions very differently in
humans than it does in panther worms. Srivastava
has likened EGR to a switch that activates other
genes involved in regeneration in panther worms,
but how this switch operates in humans remains
unclear.
Based on the texts, what would the author of Text 2
most likely say about Text 1’s characterization of the
discovery involving EGR?
A) It is reasonable given that Srivastava and her
team have identified how EGR functions in both
humans and panther worms.
B) It is overly optimistic given additional
observations from Srivastava and her team.
C) It is unexpected given that Srivastava and her
team’s findings were generally met with
enthusiasm.
D) It is unfairly dismissive given the progress that
Srivastava and her team have reported.
10
The following text is adapted from William
Shakespeare’s 1609 poem “Sonnet 27.” The poem is
addressed to a close friend as if he were physically
present.
Weary with toil, I [hurry] to my bed,
The dear repose for limbs with travel tired;
But then begins a journey in my head
To work my mind, when body’s work’s expired:
For then my thoughts—from far where I abide—
[Begin] a zealous pilgrimage to thee,
And keep my drooping eyelids open wide,
What is the main idea of the text?
A) The speaker is asleep and dreaming about
traveling to see the friend.
B) The speaker is planning an upcoming trip to the
friend’s house.
C) The speaker is too fatigued to continue a
discussion with the friend.
D) The speaker is thinking about the friend instead
of immediately falling asleep.
19
11
The following text is adapted from Lewis Carroll’s
1889 satirical novel Sylvie and Bruno. A crowd has
gathered outside a room belonging to the Warden,
an official who reports to the Lord Chancellor.
One man, who was more excited than the rest,
flung his hat high into the air, and shouted (as
well as I could make out) “Who roar for the
Sub-Warden?” Everybody roared, but whether it
was for the Sub-Warden, or not, did not clearly
appear: some were shouting “Bread!” and some
“Taxes!”, but no one seemed to know what it was
they really wanted.
All this I saw from the open window of the
Warden’s breakfast-saloon, looking across the
shoulder of the Lord Chancellor.
“What can it all mean?” he kept repeating to
himself. “I never heard such shouting
before—and at this time of the morning, too!
And with such unanimity!”
Based on the text, how does the Lord Chancellor
respond to the crowd?
A) He asks about the meaning of the crowd’s
shouting, even though he claims to know what
the crowd wants.
B) He indicates a desire to speak to the crowd, even
though the crowd has asked to speak to the
Sub-Warden.
C) He expresses sympathy for the crowd’s demands,
even though the crowd’s shouting annoys him.
D) He describes the crowd as being united, even
though the crowd clearly appears otherwise.
12
O Pioneers! is a 1913 novel by Willa Cather. In the
novel, Cather portrays Alexandra Bergson as having
a deep emotional connection to her natural
surroundings: _______
Which quotation from O Pioneers! most effectively
illustrates the claim?
A) “She had never known before how much the
country meant to her. The chirping of the insects
down in the long grass had been like the sweetest
music. She had felt as if her heart were hiding
down there, somewhere, with the quail and the
plover and all the little wild things that crooned
or buzzed in the sun. Under the long shaggy
ridges, she felt the future stirring.”
B) “Alexandra talked to the men about their crops
and to the women about their poultry. She spent
a whole day with one young farmer who had
been away at school, and who was experimenting
with a new kind of clover hay. She learned a
great deal.”
C) “Alexandra drove off alone. The rattle of her
wagon was lost in the howling of the wind, but
her lantern, held firmly between her feet, made a
moving point of light along the highway, going
deeper and deeper into the dark country.”
D) “It was Alexandra who read the papers and
followed the markets, and who learned by the
mistakes of their neighbors. It was Alexandra
who could always tell about what it had cost to
fatten each steer, and who could guess the weight
of a hog before it went on the scales closer than
John Bergson [her father] himself.”
...............................................................................................................................................................................................................................................................................................................
Unauthorized copying or reuse of any part of this page is illegal.
CO N T I N U E
20
13
Approximate Rates of Speech and
Information Conveyed for Five Languages
Language
Rate of speech
(syllables
per second)
Rate of information
conveyed (bits
per second)
Serbian 7.2 39.1
Spanish 7.7 42.0
Vietnamese 5.3 42.5
Thai 4.7 33.8
Hungarian 5.9 34.6
A group of researchers working in Europe, Asia, and
Oceania conducted a study to determine how quickly
different Eurasian languages are typically spoken
(in syllables per second) and how much information
they can effectively convey (in bits per second). They
found that, although languages vary widely in the
speed at which they are spoken, the amount of
information languages can effectively convey tends
to vary much less. Thus, they claim that two
languages with very different spoken rates can
nonetheless convey the same amount of information
in a given amount of time.
Which choice best describes data from the table that
support the researchers’ claim?
A) Among the five languages in the table, Thai and
Hungarian have the lowest rates of speech and
the lowest rates of information conveyed.
B) Vietnamese conveys information at
approximately the same rate as Spanish despite
being spoken at a slower rate.
C) Among the five languages in the table, the
language that is spoken the fastest is also the
language that conveys information the fastest.
D) Serbian and Spanish are spoken at approximately
the same rate, but Serbian conveys information
faster than Spanish does.
14
Psychologists Dacher Keltner and Jonathan Haidt
have argued that experiencing awe—a sensation of
reverence and wonder typically brought on by
perceiving something grand or powerful—can enable
us to feel more connected to others and thereby
inspire us to act more altruistically. Keltner, along
with Paul K. Piff, Pia Dietze, and colleagues, claims
to have found evidence for this effect in a recent
study where participants were asked to either gaze up
at exceptionally tall trees in a nearby grove (reported
to be a universally awe-inspiring experience) or stare
at the exterior of a nearby, nondescript building.
After one minute, an experimenter deliberately
spilled a box of pens nearby.
Which finding from the researchers’ study, if true,
would most strongly support their claim?
A) Participants who had been looking at the trees
helped the experimenter pick up significantly
more pens than did participants who had been
looking at the building.
B) Participants who helped the experimenter pick
up the pens used a greater number of positive
words to describe the trees and the building in a
postexperiment survey than did participants who
did not help the experimenter.
C) Participants who did not help the experimenter
pick up the pens were significantly more likely to
report having experienced a feeling of awe,
regardless of whether they looked at the building
or the trees.
D) Participants who had been looking at the
building were significantly more likely to notice
that the experimenter had dropped the pens than
were participants who had been looking at the
trees.
...............................................................................................................................................................................................................................................................................................................
Unauthorized copying or reuse of any part of this page is illegal.
CO N T I N U E
21
15
Employment by Sector in France and the United States, 1800–2012
(% of total employment)
Year
Agriculture
in France
Manufacturing
in France
Services
in France
Agriculture
in US
Manufacturing
in US
Services
in US
1800 64 22 14 68 18 13
1900 43 29 28 41 28 31
1950 32 33 35 14 33 53
2012 3 21 76 2 18 80
Rows in table may not add up to 100 due to rounding.
Over the past two hundred years, the percentage of the population
employed in the agricultural sector has declined in both France and the
United States, while employment in the service sector (which includes
jobs in retail, consulting, real estate, etc.) has risen. However, this
transition happened at very different rates in the two countries. This
can be seen most clearly by comparing the employment by sector in
both countries in _______
Which choice most effectively uses data from the table to complete the
statement?
A) 1900 with the employment by sector in 1950.
B) 1800 with the employment by sector in 2012.
C) 1900 with the employment by sector in 2012.
D) 1800 with the employment by sector in 1900.
Unauthorized copying or reuse of any part of this page is illegal.
CO N T I N U E
22
16
Many archaeologists will tell you that categorizing
excavated fragments of pottery by style, period,
and what objects they belong to relies not only on
standard criteria, but also on instinct developed
over years of practice. In a recent study, however,
researchers trained a deep-learning computer model
on thousands of images of pottery fragments and
found that it could categorize them as accurately as a
team of expert archaeologists. Some archaeologists
have expressed concern that they might be replaced
by such computer models, but the researchers claim
that outcome is highly unlikely.
Which finding, if true, would most directly support
the researchers’ claim?
A) In the researchers’ study, the model was able to
categorize the pottery fragments much more
quickly than the archaeologists could.
B) In the researchers’ study, neither the model nor
the archaeologists were able to accurately
categorize all the pottery fragments that were
presented.
C) A survey of archaeologists showed that
categorizing pottery fragments limits the amount
of time they can dedicate to other important
tasks that only human experts can do.
D) A survey of archaeologists showed that few of
them received dedicated training in how to
properly categorize pottery fragments.
17
Although military veterans make up a small
proportion of the total population of the
United States, they occupy a significantly higher
proportion of the jobs in the civilian government.
One possible explanation for this disproportionate
representation is that military service familiarizes
people with certain organizational structures that are
also reflected in the civilian government bureaucracy,
and this familiarity thus _______
Which choice most logically completes the text?
A) makes civilian government jobs especially
appealing to military veterans.
B) alters the typical relationship between military
service and subsequent career preferences.
C) encourages nonveterans applying for civilian
government jobs to consider military service
instead.
D) increases the number of civilian government jobs
that require some amount of military experience
to perform.
...............................................................................................................................................................................................................................................................................................................
Unauthorized copying or reuse of any part of this page is illegal.
CO N T I
N U E
23
18
Birds of many species ingest foods containing
carotenoids, pigmented molecules that are converted
into feather coloration. Coloration tends to be
especially saturated in male birds’ feathers, and
because carotenoids also confer health benefits,
the deeply saturated colors generally serve to
communicate what is known as an honest signal of
a bird’s overall fitness to potential mates. However,
ornithologist Allison J. Shultz and others have found
that males in several species of the tanager genus
Ramphocelus use microstructures in their feathers to
manipulate light, creating the appearance of deeper
saturation without the birds necessarily having to
maintain a carotenoid-rich diet. These findings
suggest that _______
Which choice most logically completes the text?
A) individual male tanagers can engage in honest
signaling without relying on carotenoid
consumption.
B) feather microstructures may be less effective
than deeply saturated feathers for signaling
overall fitness.
C) scientists have yet to determine why tanagers
have a preference for mates with colorful
appearances.
D) a male tanager’s appearance may function as
a dishonest signal of the individual’s overall
fitness.
19
When writing The Other Black Girl (2021), novelist
Zakiya Dalila Harris drew on her own experiences
working at a publishing office. The award-winning
book is Harris’s first novel, but her writing _______
honored before. At the age of twelve, she entered a
contest to have a story published in American Girl
magazine—and won.
Which choice completes the text so that it conforms
to the conventions of Standard English?
A) were
B) have been
C) has been
D) are
20
The Alvarez theory, developed in 1980 by physicist
Luis Walter Alvarez and his geologist son Walter
Alvarez, maintained that the secondary effects of an
asteroid impact caused many dinosaurs and other
animals to die
_______ it left unexplored the
question of whether unrelated volcanic activity might
have also contributed to the mass extinctions.
Which choice completes the text so that it conforms
to the conventions of Standard English?
A) out but
B) out, but
C) out
D) out,
...............................................................................................................................................................................................................................................................................................................
Unauthorized copying or reuse of any part of this page is illegal.
CO N T I
N U E
24
21
In winter, the diets of Japanese macaques, also
known as snow monkeys, are influenced more
by food availability than by food preference.
Although the monkeys prefer to eat vegetation and
land-dwelling invertebrates, those food sources may
become unavailable because of extensive snow and
ice cover,
_______ the monkeys to hunt for marine
animals in any streams that have not frozen over.
Which choice completes the text so that it conforms
to the conventions of Standard English?
A) forces
B) to force
C) forcing
D) forced
22
Lucía Michel of the University of Chile observed that
alkaline soils contain an insoluble form of iron that
blueberry plants cannot absorb, thus inhibiting
blueberry growth. If these plants were grown in
alkaline soil alongside grasses that aid in iron
solubilization, _______ Michel was determined to
find out.
Which choice completes the text so that it conforms
to the conventions of Standard English?
A) could the blueberries thrive.
B) the blueberries could thrive.
C) the blueberries could thrive?
D) could the blueberries thrive?
23
In his 1963 exhibition Exposition of Music—
Electronic Television, Korean American artist
Nam June Paik showed how television images could
be manipulated to express an artist’s perspective.
Today, Paik
_______ considered the first video artist.
Which choice completes the text so that it conforms
to the conventions of Standard English?
A) will be
B) had been
C) was
D) is
24
The first computerized spreadsheet, Dan Bricklin’s
VisiCalc, improved financial recordkeeping not only
by providing users with an easy means of adjusting
data in spreadsheets but also by automatically
updating all calculations that were dependent on
these _______ to VisiCalc’s release, changing a paper
spreadsheet often required redoing the entire sheet
by hand, a process that could take days.
Which choice completes the text so that it conforms
to the conventions of Standard English?
A) adjustments prior
B) adjustments, prior
C) adjustments. Prior
D) adjustments and prior
...............................................................................................................................................................................................................................................................................................................
Unauthorized copying or reuse of any part of this page is illegal.
CO N T I N
U E
25
25
In order to prevent nonnative fish species from
moving freely between the Mediterranean and Red
Seas, marine biologist Bella Galil has proposed that a
saline lock system be installed along the Suez Canal
in Egypt’s Great Bitter Lakes. The lock would
increase the salinity of the lakes and
_______ a
natural barrier of water most marine creatures would
be unable to cross.
Which choice completes the text so that it conforms
to the conventions of Standard English?
A) creates
B) create
C) creating
D) created
26
Despite being cheap, versatile, and easy to produce,
_______ they are made from nonrenewable
petroleum, and most do not biodegrade in landfills.
Which choice completes the text so that it conforms
to the conventions of Standard English?
A) there are two problems associated with
commercial plastics:
B) two problems are associated with commercial
plastics:
C) commercial plastics’ two associated problems are
that
D) commercial plastics have two associated
problems:
27
Stomata, tiny pore structures in a leaf that absorb
gases needed for plant growth, open when guard cells
surrounding each pore swell with water. In a pivotal
2007 article, plant cell _______ showed that lipid
molecules called phosphatidylinositol phosphates are
responsible for signaling guard cells to open stomata.
Which choice completes the text so that it conforms
to the conventions of Standard English?
A) biologist, Yuree Lee
B) biologist Yuree Lee,
C) biologist Yuree Lee
D) biologist, Yuree Lee,
28
Small, flat structures called spatulae are found at the
tips of the hairs on a spider’s leg. These spatulae
temporarily bond with the atoms of whatever they
touch. _______ spiders are able to cling to and climb
almost any surface.
Which choice completes the text with the most
logical transition?
A) For instance,
B) However,
C) Similarly,
D) As a result,
...............................................................................................................................................................................................................................................................................................................
Unauthorized copying or reuse of any part of this page is illegal.
CO N T I
N U E
26
29
In November 1934, Amrita Sher-Gil was living in
what must have seemed like the ideal city for a
young artist: Paris. She was studying firsthand the
color-saturated style of France’s modernist masters
and beginning to make a name for herself as a
painter.
_______ Sher-Gil longed to return to her
childhood home of India; only there, she believed,
could her art truly flourish.
Which choice completes the text with the most
logical transition?
A) Still,
B) Therefore,
C) Indeed,
D) Furthermore,
30
Before California’s 1911 election to approve a
proposition granting women the right to vote,
activists across the state sold tea to promote the cause
of suffrage. In San Francisco, the Woman’s Suffrage
Party sold Equality Tea at local fairs. _______ in
Los Angeles, activist Nancy Tuttle Craig, who ran
one of California’s largest grocery store firms,
distributed Votes for Women Tea.
Which choice completes the text with the most
logical transition?
A) For example,
B) To conclude,
C) Similarly,
D) In other words,
31
While researching a topic, a student has taken the
following notes:
The Seikan Tunnel is a rail tunnel in Japan.
It connects the island of Honshu to the island of
Hokkaido.
It is roughly 33 miles long.
The Channel Tunnel is a rail tunnel in Europe.
It connects Folkestone, England, to Coquelles,
France.
It is about 31 miles long.
The student wants to compare the lengths of the two
rail tunnels. Which choice most effectively uses
relevant information from the notes to accomplish
this goal?
A) Some of the world’s rail tunnels, including one
tunnel that extends from Folkestone, England, to
Coquelles, France, are longer than 30 miles.
B) The Seikan Tunnel is roughly 33 miles long,
while the slightly shorter Channel Tunnel is
about 31 miles long.
C) The Seikan Tunnel, which is roughly 33 miles
long, connects the Japanese islands of Honshu
and Hokkaido.
D) Both the Seikan Tunnel, which is located in
Japan, and the Channel Tunnel, which is located
in Europe, are examples of rail tunnels.
...............................................................................................................................................................................................................................................................................................................
Unauthorized copying or reuse of any part of this page is illegal.
CO N T I
N U E
27
32
While researching a topic, a student has taken the
following notes:
Jon Ching is a Los Angeles-based painter.
He uses the term “flauna” to describe the plant-
animal hybrids that he depicts in his surreal
paintings.
“Flauna” is a combination of the words “flora” and
“fauna.”
His painting Nectar depicts a parrot with leaves
for feathers.
His painting Primaveral depicts a snow leopard
whose fur sprouts flowers.
The student wants to provide an explanation and
example of “flauna.” Which choice most effectively
uses relevant information from the notes to
accomplish this goal?
A) The term “flauna,” used by Los Angeles-based
painter Jon Ching, is a combination of the words
“flora” and “fauna.”
B) Jon Ching uses the term “flauna,” a combination
of the words “flora” and “fauna,” to describe the
subjects of his surreal paintings: plant-animal
hybrids such as a parrot with leaves for feathers.
C) Jon Ching, who created Nectar, refers to the
subjects of his paintings as “flauna.”
D) The subjects of Nectar and Primaveral are types
of “flauna,” a term that the paintings’ creator,
Jon Ching, uses when describing his surreal
artworks.
33
While researching a topic, a student has taken the
following notes:
In the midst of the US Civil War, Susie Taylor
escaped slavery and fled to Union-army-occupied
St. Simons Island off the Georgia coast.
She began working for an all-Black army regiment
as a nurse and teacher.
In 1902, she published a book about the time she
spent with the regiment.
Her book was the only Civil War memoir to be
published by a Black woman.
It is still available to readers in print and online.
The student wants to emphasize the uniqueness of
Taylor’s accomplishment. Which choice most
effectively uses relevant information from the notes
to accomplish this goal?
A) Taylor fled to St. Simons Island, which was then
occupied by the Union army, for whom she
began working.
B) After escaping slavery, Taylor began working for
an all-Black army regiment as a nurse and
teacher.
C) The book Taylor wrote about the time she spent
with the regiment is still available to readers in
print and online.
D) Taylor was the only Black woman to publish a
Civil War memoir.
STOP
If you finish before time is called, you may check your work on this module only.
Do not turn to any other module in the test.
...............................................................................................................................................................................................................................................
Unauthorized copying or reuse of any part of this page is illegal.
28
No Test Material On This Page
Unauthorized copying or reuse of any part of this page is illegal.
CO N T I N U E
30
Module
1
Math
27
QUESTIONS
DIRECTIONS
The questions in this section address a
number
of
important
math skills.
Use
of
a calculator
is
permitted for all questions.
NOTES
Unless otherwise indicated:
All variables and expressions represent
real
numbers.
Figures provided are drawn
to
scale.
All figures lie in a plane.
The domain
of
a given function
f
is
the
set
of
all real numbers x
for
which f(x)
is
a real number.
REFERENCE
A
=nr
2
C=2nr
A=
l
w
A =
½
bh
c2
=
a2
+
b2
Special Right Triangles
V=
l
wh
V=
nr
2
h
V
=
4/3
π
r
3
V=
nr
2
h
V =
l
wh
The
number
of
degrees
of
arc in a circle
is
360.
The
number
of
radians
of
arc in a circle
is
2n.
The sum
of
the
measures in degrees
of
the
angles
of
a triangle
is
180.
Unauthorized copying or reuse of any part of this page is illegal.
CO N T I N U E
31
For multiple-choice questions, solve each problem, choose the correct
answer from
the
choices provided, and then circle your answer in this book.
Circle only one answer for each question. If you change your mind, completely
erase the circle.
You
will
not
get
credit
for
questions
with
more than one
answer circled, or
for
questions
with
no answers circled.
For student-produced response questions, solve each problem and write
your answer
next
to
or
under
the
question in
the
test
book
as
described below.
Once you've written your answer, circle
it
clearly.
You
will
not
receive credit
for
anything
written
outside
the
circle, or
for
any questions
with
more than
one circled answer.
If
you find more than one correct answer, write and circle only one answer.
Your answer
can
be
up
to
5 characters
for
a positive answer and up
to
6 characters (including the negative sign) for a negative answer,
but
no more.
If
your answer
is
a fraction
that
is
too
long (over 5 characters
for
positive,
6 characters
for
negative), write
the
decimal equivalent.
If
your answer
is
a decimal
that
is
too
long (over 5 characters for positive,
6 characters
for
negative), truncate
it
or round at the
fourth
digit.
If
your answer
is
a mixed number (such
as
3
½
2
),
write
it
as
an
improper
fraction (7/2) or its decimal equivalent
(3
.
5)
.
Don't
include symbols such
as
a percent sign, comma, or dollar sign in
your circled answer.
1
What is 10% of 470?
A) 37
B) 47
C) 423
D) 460
2
4x + 6 = 18
Which equation has the same solution as the given
equation?
A)
4x = 108
B)
4x = 24
C)
4x = 12
D)
4x = 3
3
The total cost, in dollars, to rent a surfboard consists
of a
$25
service fee and a
$10
per hour rental fee. A
person rents a surfboard for t hours and intends to
spend a maximum of
$75
to rent the surfboard.
Which inequality represents this situation?
A)
t10 75
B)
10 + 25t 75
C)
25t 75
D)
25 + 10t 75
4
The function g is defined by
g x x= + 9
2
( )
. For
which value of x is
g(x) = 25
?
A) 4
B) 5
C) 9
D) 13
5
Each face of a fair 14-sided die is labeled with a
number from 1 through 14, with a different number
appearing on each face. If the die is rolled one time,
what is the probability of rolling a 2?
A)
1
14
B)
2
14
C)
12
14
D)
13
14
6
A printer produces posters at a constant rate of
42 posters per minute. At what rate, in posters
per hour, does the printer produce the posters?
...............................................................................................................................................................................................................................................................................................................
Unauthorized copying or reuse of any part of this page is illegal.
CO N T I N U E
32
7
The function f is defined by the equation
f x x= 7 + 2( )
. What is the value of
f x( )
when
x = 4
?
8
A teacher is creating an assignment worth 70 points.
The assignment will consist of questions worth
1 point and questions worth 3 points. Which
equation represents this situation, where x represents
the number of 1-point questions and y represents the
number of 3-point questions?
A)
xy4 = 70
B)
x y4( + ) = 70
C)
x y3 + = 70
D)
x y+ 3 = 70
9
Right triangles
LMN
and
PQR
are similar, where L
and M correspond to P and Q, respectively. Angle M
has a measure of
53°
. What is the measure of
angle Q ?
A)
37°
B)
53°
C)
127°
D)
143°
10
y x= -3
x y4 + = 15
The solution to the given system of equations
is
(x, y)
. What is the value of x ?
A) 1
B) 5
C) 15
D) 45
11
Which of the following equations is the most
appropriate linear model for the data shown in the
scatterplot?
A)
y
x
=
-1.9 - 10.1
B)
y
x
=
-1.9 + 10.1
C)
y
x
= 1.9 - 10.1
D)
y
x
=
1.9 + 10.1
...............................................................................................................................................................................................................................................................................................................
Unauthorized copying or reuse of any part of this page is illegal.
CO N T I N U E
33
12
The graph of
y
f x= ( )
is shown, where the
function f is defined by
f
x ax bx cx d( )
= + + +
3 2
and a, b, c, and d are constants. For how many
values of x does
f
x = 0( )
?
A) One
B) Two
C) Three
D) Four
13
Vivian bought party hats and cupcakes for $71. Each
package of party hats cost $3, and each cupcake cost
$1. If Vivian bought 10 packages of party hats, how
many cupcakes did she buy?
14
z
z+ 10 - 24 = 0
2
What is one of the solutions to the given equation?
15
Bacteria are growing in a liquid growth medium.
There were 300,000 cells per milliliter during an
initial observation. The number of cells per milliliter
doubles every 3 hours. How many cells per milliliter
will there be 15 hours after the initial observation?
A) 1,500,000
B) 2,400,000
C) 4,500,000
D) 9,600,000
16
Which expression is equivalent to
x
y x y6 + 12
8
2 2 2
?
A)
x
y x6 2
2 2 6
( )
B)
x
y x6
2 2 4
( )
C)
x
y x6 + 2
2 2 6
(
)
D)
x
y x6 + 2
2 2 4
(
)
...............................................................................................................................................................................................................................................................................................................
Unauthorized copying or reuse of any part of this page is illegal.
CO N T I N U E
34
17
A neighborhood consists of a 2-hectare park and a
35-hectare residential area. The total number of
trees in the neighborhood is 3,934. The equation
x y2 + 35 = 3,934
represents this situation. Which of
the following is the best interpretation of x in this
context?
A) The average number of trees per hectare in
the park
B) The average number of trees per hectare in
the residential area
C) The total number of trees in the park
D) The total number of trees in the residential area
18
The graph shows the relationship between the
number of shares of stock from Company A, x, and
the number of shares of stock from Company B, y,
that Simone can purchase. Which equation could
represent this relationship?
A)
y
x= 8 + 12
B)
x
y8 + 12 = 480
C)
y
x= 12 + 8
D)
x
y12 + 8 = 480
19
Circle A has a radius of
3
n
and circle B has a radius
of
n
129
, where n is a positive constant. The area of
circle
B is how many times the area of circle A?
A) 43
B) 86
C) 129
D) 1,849
...............................................................................................................................................................................................................................................................................................................
Unauthorized copying or reuse of any part of this page is illegal.
CO N T I N U E
35
20
Data value Frequency
63
73
88
98
10 9
11 11
12 9
13 0
14 6
The frequency table summarizes the 57 data values in
a data set. What is the maximum data value in the
data set?
21
A circle in the
xy
-plane has a diameter with
endpoints
2, 4()
and
()2, 14
. An equation of this
circle is
xyr(−2) + ( 9) =
222
, where r is a positive
constant. What is the value of r ?
22
The measure of angle R is
π2
3
radians. The measure
of angle T is
π5
12
radians greater than the measure of
angle R. What is the measure of angle T, in degrees?
A) 75
B) 120
C) 195
D) 390
23
A certain town has an area of
4.36
square miles.
What is the area, in square yards, of this town?
(1 mile = 1,760 yards)
A) 404
B) 7,674
C) 710,459
D) 13,505,536
...............................................................................................................................................................................................................................................................................................................
Unauthorized copying or reuse of any part of this page is illegal.
CONTINUE
36
24
x y
18 130
23 160
26 178
For line h, the table shows three values of x and their
corresponding values of y. Line k is the result of
translating line h down 5 units in the
xy
-plane.
What is the x-intercept of line k?
A)
26
3
, 0
B)
9
2
, 0
C)
11
3
, 0
D)
17
6
, 0
25
In the xy-plane, the graph of the equation
y x x= + 9 100
2
intersects the line
y c=
at
exactly one point. What is the value of c?
A)
481
4
B)
−100
C)
319
4
D)
9
2
...............................................................................................................................................................................................................................................................................................................
Unauthorized copying or reuse of any part of this page is illegal.
CO N T I N U E
37
26
x y2 + 3 = 7
x y10 + 15 = 35
For each real number r, which of the following
points lies on the graph of each equation in the
xy-plane for the given system?
A)
r r
5
+ 7,
5
+ 35
B)
r
r
3
2
+
7
2
,
C)
r
r
,
2
3
+
7
3
D)
r
r
,
3
2
+
7
2
27
The perimeter of an equilateral triangle is
624 centimeters. The height of this triangle is
k 3
centimeters, where k is a constant. What is the
value of k?
STOP
If you finish before time is called, you may check your work on this module only.
Do not turn to any other module in the test.
.................................................................................................................................................................................................................................................
Unauthorized copying or reuse of any part of this page is illegal.
38
No Test Material On This Page
Unauthorized copying or reuse of any part of this page is illegal.
CO N T I N U E
40
Module
2
Math
27
QUESTIONS
DIRECTIONS
The questions in this section address a
number
of
important
math skills.
Use
of
a calculator
is
permitted for all questions.
NOTES
Unless otherwise indicated:
All variables and expressions represent
real
numbers.
Figures provided are drawn
to
scale.
All figures lie in a plane.
The domain
of
a given function
f
is
the
set
of
all real numbers
x
for
which
f(x)
is
a real number.
REFERENCE
A
=nr
2
C=2nr
A=
l
w
A =
½
bh
c2
=
a2
+
b2
Special Right Triangles
V=
l
wh
V=
nr
2
h
V
=
4/3
π
r
3
V=
π
r
2
h
V =
1
3
l
wh
The
number
of
degrees
of
arc in a circle
is
360.
The
number
of
radians
of
arc in a circle
is
2n.
The sum
of
the
measures in degrees
of
the
angles
of
a triangle
is
180.
Unauthorized copying or reuse of any part of this page is illegal.
CO N T I N U E
41
For multiple-choice questions, solve each problem, choose the correct
answer from
the
choices provided, and then circle your answer in this book.
Circle only one answer for each question. If you change your mind, completely
erase the circle.
You
will
not
get
credit
for
questions
with
more than one
answer circled, or
for
questions
with
no answers circled.
For student-produced response questions, solve each problem and write
your answer
next
to
or
under
the
question in
the
test
book
as
described below.
Once you've written your answer, circle
it
clearly.
You
will
not
receive credit
for
anything
written
outside
the
circle, or
for
any questions
with
more than
one circled answer.
If
you find more than one correct answer, write and circle only one answer.
Your answer
can
be
up
to
5 characters
for
a positive answer and up
to
6 characters (including the negative sign) for a negative answer,
but
no more.
If
your answer
is
a fraction
that
is
too
long (over 5 characters
for
positive,
6 characters
for
negative), write
the
decimal equivalent.
If
your answer
is
a decimal
that
is
too
long (over 5 characters for positive,
6 characters
for
negative), truncate
it
or round at the
fourth
digit.
If
your answer
is
a mixed number (such
as
3
½
2
),
write
it
as
an
improper
fraction (7/2) or its decimal equivalent
(3
.
5)
.
Don't
include symbols such
as
a percent sign, comma, or dollar sign in
your circled answer.
1
Tilly earns p dollars for every w hours of work.
Which expression represents the amount of money,
in dollars, Tilly earns for
w39
hours of work?
A)
p39
B)
p
39
C)
p + 39
D)
p - 39
2
For a training program, Juan rides his bike at an
average rate of 5.7 minutes per mile. Which
function m models the number of minutes it will take
Juan to ride x miles at this rate?
A)
m x
x
=
5.7
( )
B)
m x x= + 5.7( )
C)
m x x= - 5.7( )
D)
m x x= 5.7( )
3
x
3
= 12
x y−3 + = 6
The solution to the given system of equations
is
x y( , )
. What is the value of y ?
A)
−3
B)
6
C)
18
D)
30
4
s t= 40 + 3
The equation gives the speed s, in miles per hour, of
a certain car t seconds after it began to accelerate.
What is the speed, in miles per hour, of the car
5 seconds after it began to accelerate?
A) 40
B) 43
C) 45
D) 55
...............................................................................................................................................................................................................................................................................................................
Unauthorized copying or reuse of any part of this page is illegal.
CO N T I N U E
42
5
Note: Figure
not
drawn
to
scale.
For the right triangle shown,
a
= 4
and
b
= 5
.
Which expression represents the value of c
?
A)
4
+ 5
B)
(
4)(5)
C)
4
+ 5
D)
4
+ 5
2 2
6
x
4 + 5 = 165
What is the solution to the given equation?
7
The x-intercept of the graph shown is
x
(
,
0)
. What is
the value of x ?
8
The function f is defined by
f
x x=
1
10
2( )
. What
is the y-intercept of the graph of
y
f x= ( )
in the
xy-plane?
A)
2, 0( )
B)
0
, −2( )
C)
0
,
1
1
0
D)
1
1
0
,
0
...............................................................................................................................................................................................................................................................................................................
Unauthorized copying or reuse of any part of this page is illegal.
CO N T I N U E
43
9
The function f is defined by
fx x() = 7
3
.
In
the
xy-plane, the graph of
yg x=( )
is the result of
shifting the graph of
yf x=( )
down 2 units. Which
equation defines function g ?
A)
gx x() =
7
2
3
B)
gx x() = 7
3
2
C)
gx x() = 7 + 2
3
D)
gx x() = 7 2
3
10
x +7 = 10
xy+7 =
2
()
Which ordered pair
xy,()
is a solution to the given
system of equations?
A)
3, 100()
B)
3, 3()
C)
3, 10()
D)
3, 70()
11
Which expression is equivalent
to
xx x x7+ 7 6 3
33
() ()
?
A)
xx+10
3
B)
xx−13 + 10
3
C)
xx−13 + 4
3
D)
xx+4
3
12
The function p is defined by
pn n=7
3
()
. What is the
value of n when
pn()
is equal to 56?
A)
2
B)
8
3
C)
7
D)
8
...............................................................................................................................................................................................................................................................................................................
Unauthorized copying or reuse of any part of this page is illegal.
CONTINUE
44
13
Note: Figure
not
drawn
to
scale.
In the figure shown, line c intersects parallel lines s
and t. What is the value of x ?
14
A list of 10 data values is shown.
6, 8, 16, 4, 17, 26, 8, 5, 5, 5
What is the mean of these data?
15
The equation
E
t( ) = 5(1.8)
t
gives the estimated
number of employees at a restaurant, where t is the
number of years since the restaurant opened. Which
of the following is the best interpretation of the
number 5 in this context?
A) The estimated number of employees when the
restaurant opened
B) The increase in the estimated number of
employees each year
C) The number of years the restaurant has been
open
D) The percent increase in the estimated number of
employees each year
16
g
x x= + 55
2
( )
What is the minimum value of the given function?
A) 0
B) 55
C) 110
D) 3,025
...............................................................................................................................................................................................................................................................................................................
Unauthorized copying or reuse of any part of this page is illegal.
CO N T I N U E
45
17
Each year, the value of an investment increases by
0.49% of its value the previous year. Which of the
following functions best models how the value of the
investment changes over time?
A) Decreasing exponential
B) Decreasing linear
C) Increasing exponential
D) Increasing linear
18
The population of Greenville increased by
7%
from 2015 to 2016. If the 2016 population is k times
the 2015 population, what is the value of k ?
A) 0.07
B) 0.7
C) 1.07
D) 1.7
19
Which expression is equivalent to
a
11
12
,
where
a > 0
?
A)
a
13212
B)
a
132144
C)
a
132121
D)
a
13211
20
An event planner is planning a party. It costs the
event planner a onetime fee of
$35
to rent the venue
and
$10.25
per attendee. The event planner has a
budget of
$200
. What is the greatest number of
attendees possible without exceeding the budget?
21
If
x4 4 = 112
, what is the positive value of
x 1
?
22
A cube has an edge length of 68 inches. A solid
sphere with a radius of 34 inches is inside the cube,
such that the sphere touches the center of each face
of the cube. To the nearest cubic inch, what is the
volume of the space in the cube not taken up by the
sphere?
A) 149,796
B) 164,500
C) 190,955
D) 310,800
...............................................................................................................................................................................................................................................................................................................
Unauthorized copying or reuse of any part of this page is illegal.
CO N T I N U E
46
23
What is the diameter of the circle in the
xy
-plane
with equation
x y( 5) + ( 3) = 16
2 2
?
A) 4
B) 8
C) 16
D) 32
24
For the exponential function f, the value of
f (1)
is k,
where k is a constant. Which of the following
equivalent forms of the function f shows the value of
k as the coefficient or the base?
A)
f x( ) = 50(1.6)
x+1
B)
f x( ) = 80(1.6)
x
C)
f x( ) = 128(1.6)
x1
D)
f x( ) = 204.8(1.6)
x2
25
A model estimates that at the end of each year
from 2015 to 2020, the number of squirrels in a
population was
150%
more than the number of
squirrels in the population at the end of the previous
year. The model estimates that at the end of 2016,
there were 180 squirrels in the population. Which of
the following equations represents this model, where
n is the estimated number of squirrels in the
population t years after the end of 2015 and
t 5
?
A)
n = 72(1.5)
t
B)
n = 72(2.5)
t
C)
n = 180(1.5)
t
D)
n = 180(2.5)
t
...............................................................................................................................................................................................................................................................................................................
Unauthorized copying or reuse of any part of this page is illegal.
CO N T I N U E
47
26
x y5 + 7 = 1
ax by+ = 1
In the given pair of equations, a and b are constants.
The graph of this pair of equations in the
xy-plane is
a pair of perpendicular lines. Which of the following
pairs of equations also represents a pair of
perpendicular lines?
A)
x y10 + 7 = 1
ax by 2 = 1
B)
x y10 + 7 = 1
ax by+ 2 = 1
C)
x y10 + 7 = 1
ax by2 + = 1
D)
x y5 7 = 1
ax by+ = 1
27
x x c 34 + = 0
2
In the given equation, c is a constant. The equation
has no real solutions if
c n >
. What is the least
possible value of n?
STOP
If you finish before time is called, you may check your work on this module only.
Do not turn to any other module in the test.
...............................................................................................................................................................................................................................................
Unauthorized copying or reuse of any part of this page is illegal.
48
No Test Material On This Page
No
Test
Material
On
This
Page
No
Test
Material
On
This
Page
No
Test
Material
On
This
Page
No
Test
Material
On
This
Page
The
SAT
®
GENERAL
DIRECTIONS
You
may work on only one module
at
a time.
If you finish a module before time is called,
check
your
work
on
that
module only.
You
may
NOT
turn
to
any
other
module.
MARKING
YOUR ANSWERS
Be
sure
to
answer your questions properly in this
book
.
Circle only one answer
to
each question. If you change your mind, completely
erase the circle.
You
will
not
get
credit
for
questions with more than one answer
circled,
or
for
questions with no answers circled.
USING
YOUR
TEST
BOOK
You
may use
the
test
book
for
scratch work.
You
may
not
fold
or
remove pages
or
portions
of
a page from this book,
or
take the
book
from
the
testing room.
© 2022 College Board. College Board, SAT, and the acorn logo are registered trademarks of College Board.
WF2P0001
The SAT
®
Practice
Test
#
1
ANSWER EXPLANATIONS
These answer explanations are for students taking the
digital SAT in nondigital format.
© 2022 College Board. College Board, SAT, and the acorn logo are registered trademarks of College Board.
SAT ANSWER EXPLANATIONS
n
READING AND WRITING: MODULE 1
2 SAT PRACTICE TEST #1 ANSWER EXPLANATIONS
Reading and Writing
Module 1
(33 questions)
QUESTION 1
Choice B is the best answer because it most logically completes the text’s
discussion of Ochoa’s prediction that humans will one day need to live in places
other than Earth. As used in this context, “speculates” would mean puts forward
an idea without firm evidence. The text states that Ochoa “doesn’t have a
definite idea” about when humans might need to live in other environments and
characterizes Ochoa’s prediction as a “conjecture,” or a conclusion presented
without convincing evidence. This context indicates that Ochoa speculates when
she makes this prediction.
Choice A is incorrect because saying that Ochoa “demands,” or insists or
requires, that humans will one day need to live in other environments than Earth’s
wouldn’t make sense in context. The text indicates that she’s unsure about
the timing but hypothesizes that it will someday happen. Choice C is incorrect
because saying that Ochoa “doubts,” or questions or disbelieves, that humans
will one day need to live in other environments than Earth’s wouldn’t make sense
in context. The text indicates that although Ochoa is unsure about the timing,
she hypothesizes that humans will need to live in places other than Earth and
encourages research into future travel to the moon. Choice D is incorrect because
saying that Ochoa “establishes,” or proves, that humans will one day need to
live in other environments than Earth’s wouldn’t make sense in context. Rather
than stating that Ochoa discusses her idea with certainty and supports it with
evidence, the text indicates that Ochoa is unsure about when humans might need
to live in other environments.
SAT ANSWER EXPLANATIONS
n
READING AND WRITING: MODULE 1
3 SAT PRACTICE TEST #1 ANSWER EXPLANATIONS
QUESTION 2
Choice C is the best answer because it most logically completes the text’s
discussion of Annie Dodge Wauneka’s work as a Navajo Nation legislator. As used
in this context, “persistent” means existing continuously. The text states that
Wauneka “continuously worked to promote public health,” traveling extensively
and authoring a medical dictionary; this indicates that Wauneka’s effort was
persistent.
Choice A is incorrect because describing Wauneka’s effort related to public
health as “impartial,” or not partial or biased and treating all things equally,
wouldn’t make sense in context. The text suggests that Wauneka’s continuous
work was partial in one way, as she focused specifically on promoting public
health throughout the Navajo homeland and to speakers of the Navajo language.
Choice B is incorrect because the text emphasizes that Wauneka’s effort to
promote public health as a Navajo Nation legislator was continuous and extensive,
involving wide travels and the authoring of a medical dictionary. Because this
work clearly involved care and dedication, it wouldn’t make sense to describe it
as “offhand,” or casual and informal. Choice D is incorrect because nothing in the
text suggests that Wauneka’s effort to promote public health was “mandatory,”
or required by law or rule, even though Wauneka was a Navajo Nation legislator.
Rather than suggesting that Wauneka’s effort was required for any reason, the
text emphasizes the continuous and extensive nature of her work.
QUESTION 3
Choice D is the best answer because it most logically completes the text’s
discussion of the collaboration between the Crow Tribe and Montana State
University. As used in this context, “exemplifies” means demonstrates. The text
conveys how the Crow Tribe–Montana State University collaboration serves to
illustrate the model of community-based participatory research introduced earlier
in the text and expanded on later in the text.
Choice A is incorrect because referring to “circumvents,” or avoids, wouldn’t make
sense in context. The text suggests that the Crow Tribe–Montana State University
collaboration serves as an example of the principles of community-based
participatory research, not that the collaboration evades this model. ChoiceB
is incorrect because referring to “eclipses,” or overshadows, wouldn’t make
sense in context. The text describes the Crow Tribe–Montana State University
collaboration as an equal partnership, which indicates that it’s an example of the
community-based participatory research model, not that it overshadows the
model. Choice C is incorrect because saying that the collaboration “fabricates,”
or creates, the model wouldn’t make sense in context. The text indicates that the
Crow Tribe–Montana State University collaboration serves as an example of the
model, not that it created the model.
SAT ANSWER EXPLANATIONS
n
READING AND WRITING: MODULE 1
4 SAT PRACTICE TEST #1 ANSWER EXPLANATIONS
QUESTION 4
Choice A is the best answer because it most logically completes the text’s
discussion of a relationship between the dodder plant and its host plant. As used
in this context, “synchronization” means the act of things happening at the same
time. The text indicates that the dodder and its host plant flower in unison and that
this synchronization occurs because the dodder makes use of a protein produced
by the host shortly before flowering.
Choice B is incorrect because referring to “hibernation,” or the state of being
dormant or inactive, wouldn’t make sense in context. The text focuses on
something the dodder plant actively engages in—making use of a protein and
producing flowers. Choice C is incorrect because stating that the dodder plant
and its host engage together in “prediction,” or the act of declaring or indicating
something in advance, wouldn’t make sense in context. Rather than indicating
that the dodder plant and its host plant make a prediction about flowering activity,
the text suggests that the host produces a protein as part of its regular flowering
process and that the dodder then absorbs and uses that protein to flower at the
same time. Choice D is incorrect because referring to “moderation,” or the act
of causing something to become less intense or extreme, wouldn’t make sense
in context. Although the text states that the dodder plant absorbs and uses a
protein made by its host plant, it doesn’t suggest that the dodder lessens the host
plant’s flowering activity; the two plants simply flower in unison.
QUESTION 5
Choice B is the best answer because it most logically completes the text’s
description of efforts to explain the existence of planets in binary star systems.
As used in this context, describing an explanation as “a straightforward” one
would mean that the explanation is direct and uncomplicated. The text asserts
that since it should be “nearly impossible” for planets to form in binary star
systems, it’s “not surprising” that there isn’t a straightforward explanation for
the existence of planets in such systems; the fact that one potential approach
involves “complex” factors offers further contextual support for this idea.
Choice A is incorrect because it would not make sense in context to say that
there isn’t “a discernible” explanation—meaning an explanation capable of being
perceived—for the existence of planets in binary star systems. The text discusses
just such an explanation offered by Roman Rafikov and Kedron Silsbee, which
indicates that their explanation can be discerned. Choice C is incorrect because
the text emphasizes how difficult it is to explain the existence of planets in
binary star systems, suggesting that the situation isn’t marked by the lack of “an
inconclusive” explanation—an explanation that does not resolve the issue—but
rather that if any explanations have been offered, they’ve likely been inconclusive
ones. Choice D is incorrect because nothing in the text suggests that there is a
lack of “an unbiased,” or impartial and unprejudiced, explanation for the existence
of planets in binary star systems. The text indicates that it’s difficult to explain the
existence of planets in such systems and it describes one attempt to do so, but
there is no evidence that explanations from Roman Rafikov and Kedron Silsbee or
others are biased.
SAT ANSWER EXPLANATIONS
n
READING AND WRITING: MODULE 1
5 SAT PRACTICE TEST #1 ANSWER EXPLANATIONS
QUESTION 6
Choice A is the best answer because it most logically completes the text’s
discussion of Sterlin Harjo’s approach to representing Native characters on
television. As used in this context, “repudiates” means rejects or refuses to have
anything to do with. The text indicates that television shows tend to depict Native
characters as living long ago, but that Harjo’s series Reservation Dogs focuses
on Native teenagers in the present day, representing a “rejection” of the typical
approach to depicting Native characters. This context thus indicates that Harjo
repudiates television’s general tendency regarding Native characters.
Choice B is incorrect because the text describes Harjo’s “rejection” of the typical
approach to representing Native characters on television, so it wouldn’t make sense
to say that Harjo “proclaims,” or declares or affirms, television’s general tendency
regarding Native characters. Harjo is described as refusing to follow the pattern
of depicting Native characters in the distant past, not as proclaiming that pattern.
Choice C is incorrect because the text describes television’s tendency to represent
Native characters in the distant past as something that is already occurring, not
as something that Harjo “foretells,” or predicts will happen in the future. The text is
focused on Harjo’s “rejection” of this pattern, not on any predictions he may have
about it. Choice D is incorrect because saying that Harjo “recants” something would
mean that he withdraws a previously held belief, and it wouldn’t make sense to say
that Harjo recants television’s tendency to represent Native characters as living in
the past. No beliefs previously held by Harjo are mentioned. Additionally, a tendency
isn’t a belief and thus isn’t something that can be recanted.
QUESTION 7
Choice A is the best answer because it most accurately states the main purpose
of the text. After providing a brief introduction to computer scientist Luis von
Ahn, the text focuses on discussing how von Ahn’s digitization work led to the
invention of a digital security test known as reCAPTCHA.
Choice B is incorrect because the text doesn’t address how digital scanners
work. Choice C is incorrect. Although the text mentions von Ahn’s book-digitizing
project, that information is provided as a detail, not as the main purpose of the
text. Choice D is incorrect because the text doesn’t provide any indication of
reCAPTCHA’s popularity; instead, it describes reCAPTCHA’s origin.
QUESTION 8
Choice D is the best answer because it best describes how the underlined
sentence functions in the text as a whole. The first sentence of the text establishes
that Lily can be “keenly sensitive to” scenes that serve as a “fitting background”
for her feelings—that is, she’s very aware of when a setting seems to reflect her
mood. The next sentence, which is underlined, then demonstrates this awareness:
Lily views the landscape she’s in as a large-scale reflection of her current
mood, identifying with elements such as its calmness. Thus, the function of the
underlined sentence is to illustrate an idea introduced in the previous sentence.
SAT ANSWER EXPLANATIONS
n
READING AND WRITING: MODULE 1
6 SAT PRACTICE TEST #1 ANSWER EXPLANATIONS
Choice A is incorrect because the underlined sentence describes the scene
only in very general terms, referring to its calmness, breadth, and long stretches
of land. It’s the next sentence that adds specific details about colors, light, and
various trees nearby. Choice B is incorrect because nothing in the underlined
sentence suggests that Lily is experiencing an internal conflict. In fact, the
sentence indicates that Lily thinks the landscape reflects her own feeling of
calmness. Choice C is incorrect because the only assertion in the underlined
sentence is that Lily feels that broad aspects of the landscape, such as its
calmness, reflect her current mood, and that assertion isn’t expanded on in the
next sentence. Instead, the next sentence describes specific details of the scene
without connecting them to Lily’s feelings.
QUESTION 9
Choice C is the best answer because it best describes how the underlined
sentence functions in the text as a whole. The first sentence presents the
implications of Veeraraghavan’s team’s study: sunshine exposure during work
hours can cause overly optimistic behavior. The underlined sentence then
describes the data the team consulted and how they were used (comparing
predictions about earnings to what the companies actually earned), and the final
sentence presents what the team found in their examination of the data. Thus, the
underlined sentence mainly functions to explain part of the methodology used in
the team’s study.
Choice A is incorrect because the underlined sentence explains in part how the
team conducted their analysis of the effect of sunshine but doesn’t address what
the team found; a broad summary is instead given in the other two sentences.
Choice B is incorrect because the underlined sentence doesn’t present any
specific examples from the team’s comparisons of 29,000 earnings predictions to
actual earnings; it simply explains in part how the team conducted their analysis.
Choice D is incorrect because the underlined sentence simply explains in part
how the team conducted their analysis; the text never mentions any challenges
that the team encountered in their study.
QUESTION 10
Choice C is the best answer because it describes something that is true of
Mother, as presented in the text. The text indicates that in addition to other
activities, Mother writes stories for her children while they are at school and
makes up “funny pieces of poetry” for certain occasions.
Choice A is incorrect because the text suggests that Mother prefers to spend
her time with her children and doesn’t sit at home hoping that ladies will visit her.
Choice B is incorrect because the text says only that Mother makes up poetry
for the children’s birthdays, not that she likes birthdays more than other special
occasions. Choice D is incorrect because the text doesn’t suggest that Mother
prefers reading to her children over the other activities she does with them, such
as playing with them and writing stories and poems for them.
SAT ANSWER EXPLANATIONS
n
READING AND WRITING: MODULE 1
7 SAT PRACTICE TEST #1 ANSWER EXPLANATIONS
QUESTION 11
Choice A is the best answer because it most accurately states the main purpose
of the text. In the first part of the text, the speaker addresses Paul Laurence
Dunbar’s ability to understand people (he has “read the hearts and souls of men”
and written of their “joy and mirth”). In the second part of the text, the speaker
describes Dunbar’s thorough understanding of the natural world (he has read “the
language of the flowers” and engaged with “the little brook”). Thus, the text mainly
praises Dunbar for being especially perceptive about people and nature.
Choice B is incorrect because the speaker describes Dunbar as having read the
“hearts and souls of men” and the “language of flowers” to convey Dunbar’s ability
to comprehend people and nature, not to suggest that Dunbar has literally read
any of these things or has read a great deal about them. Choice C is incorrect
because the text notes how well Dunbar has made sense of the topics he’s written
about but doesn’t address any specific parts of Dunbar’s writing process beyond
the suggestion that he used a pen. Choice D is incorrect because the text focuses
on Dunbar’s understanding of people and nature as expressed in his writing.
Nothing in the text suggests that the speaker is recalling a particular afternoon
actually spent in nature with Dunbar; even if there had been a shared experience,
the text isn’t focused on reminiscing.
QUESTION 12
Choice A is the best answer because it presents the quotation that most directly
illustrates the claim that Whitman’s poem suggests that its readers haven’t fully
understood themselves. This quotation makes that point directly by saying to
readers, “You have not known what you are.” The quotation goes on to reinforce
this point using a metaphor of sleep, saying that readers have “slumber’d” and
that their “eyelids have been the same as closed most of the time.”
Choice B is incorrect because this quotation doesn’t suggest that readers haven’t
fully understood themselves but instead characterizes readers as “immense”
and “interminable.” Although immense and interminable things can be difficult to
understand, this quotation doesn’t make that point. Choice C is incorrect because
this quotation doesn’t suggest that readers haven’t fully understood themselves
but instead conveys the speaker’s regret over not having celebrated readers
sooner. In fact, this quotation says nothing at all about readers themselves—it’s
focused solely on the speaker’s feelings about readers. Choice D is incorrect
because this quotation doesn’t suggest that readers haven’t fully understood
themselves; instead, this quotation makes the point that the speaker has
understood readers and is determined to create “hymns” about them.
SAT ANSWER EXPLANATIONS
n
READING AND WRITING: MODULE 1
8 SAT PRACTICE TEST #1 ANSWER EXPLANATIONS
QUESTION 13
Choice A is the best answer because it presents a finding that, if true, would
support the claim about Chambi’s photographs. The text describes a student
advancing the claim that Chambi’s photographs “have considerable ethnographic
value”—meaning that they are valuable as records of cultures—and that they
capture diverse elements of Peruvian society” in a respectful way. If it’s true that
Chambi carefully photographed people from a range of different communities in
Peru as well as photographed the customs and sites of different communities,
that would lend support to the claim that the photographs have ethnographic
value as depictions of diverse elements of society in Peru.
Choice B is incorrect because the student’s claim is that Chambi’s photographs
have considerable ethnographic value because they depict diverse elements
of Peruvian society; the student doesn’t claim anything about the technical skill
demonstrated in the photographs. Choice C is incorrect because neither Chambi’s
reputation nor the locations where his photographs may have been published
would be relevant to the student’s claim that his photographs are valuable as an
ethnographic record of Peru’s diverse society. Choice D is incorrect because
the popularity among other photographers of the people and places that
Chambi photographed would be irrelevant to the student’s claim that Chambi’s
photographs are valuable as an ethnographic record of Peru’s diverse society.
QUESTION 14
Choice C is the best answer because it uses data from the table to effectively
exemplify the idea that the film outputs of the four individuals included in the table
should be considered bare minimums—that is, that we should assume that the
individuals actually had higher outputs than those recorded. The table presents the
years during which the individuals were active and the number of known films the
individuals are credited in. The table indicates that Lillian St. Cyr has 66 film credits
as an actor and that Edwin Carewe has 58 film credits as a director; it follows that if
some films and records for the era were lost, it’s possible that Lillian St. Cyr acted
in far more than 66 films and that Edwin Carewe directed more than 58 films.
Choice A is incorrect because it doesn’t effectively exemplify the idea that the
film outputs of the four individuals included in the table should be considered
bare minimums. Rather than addressing the idea that the individuals likely had
higher outputs than those presented in the table, this choice simply compares
data from the table to make the point that Dark Cloud has fewer credited acting
roles than Lillian St. Cyr (35 and 66, respectively). Choice B is incorrect because
it misrepresents data from the table, even though it may exemplify the idea that
the film outputs of the four individuals included in the table should be considered
bare minimums by implying that Edwin Carewe acted in more than 47 films. The
table indicates that Edwin Carewe was active from 1912 to 1934, meaning that his
47 credited acting roles were in films made before or during 1934, not after that
time. Choice D is incorrect because it doesn’t effectively exemplify the idea that
the film outputs of the four individuals included in the table should be considered
SAT ANSWER EXPLANATIONS
n
READING AND WRITING: MODULE 1
9 SAT PRACTICE TEST #1 ANSWER EXPLANATIONS
bare minimums. Instead of addressing the idea that the individuals likely had
higher outputs than those recorded, this choice suggests that James Young
Deer actually acted in and directed fewer films than presented in the table (only
33 known films as a director instead of 35, and only 10 known films as an actor
instead of 33).
QUESTION 15
Choice D is the best answer because it provides the most direct support from the
table for the claim that the plants growing in close proximity to other plants gained
an advantage at an early developmental stage. The table shows the total number
of juvenile plants from five species that were found growing on bare ground and
in patches of vegetation as well as the percentage of the total number of each
species that were growing in patches of vegetation. For each of the five species,
more than 50% of the juvenile plants were growing in patches of vegetation. The
text notes, however, that a random distribution of plants across the landscape
should result in only about 15% of the plants being found in patches of vegetation.
In other words, for each of the five species, the percentage of juvenile plants found
growing in patches of vegetation was substantially higher than could be explained
by chance alone. This finding supports the claim in the text: if plants growing in
patches are overrepresented among plants that have survived to the juvenile
stage, as the data show they are, then it suggests that it’s advantageous for plants
at an early stage of development to grow in patches of vegetation.
Choice A is incorrect because the statement that less than 75% of juvenile plants
were found growing in patches of vegetation, while true, doesn’t clearly support
the claim that the plants growing in close proximity to other plants gained an
advantage at an early developmental stage. Saying that less than 75% of plants
were found in patches doesn’t indicate how the percentage growing in patches
compares with the percentage that would be expected to grow in patches on the
basis of chance alone, which is the information necessary to evaluate whether
the claim in the text has support in the table. Put another way, if the percentage
of plants found growing in patches was 15% or less, it would be true that less
than 75% were found in patches, but the data would in fact weaken the claim in
the text, not strengthen it, since the data would show that growing in patches
wasn’t advantageous. Choice B is incorrect because only 12 plants of this species
were found growing in patches, which was the lowest number of any species, not
the greatest number. Additionally, even if it were true that this species had the
greatest number of plants growing in patches, the finding would be irrelevant to
the claim that plants of all five species gained an advantage by growing in close
proximity to other plants. Choice C is incorrect because 59.1% of the plants of
these species were found growing in patches, which is a far greater percentage,
not a lower percentage, than what would be expected if plants were randomly
distributed (around 15%). Additionally, if it were true that the percentage of plants
growing in patches was lower for these species than what would be expected
from chance alone, that finding would weaken, not strengthen, the claim that
growing in patches is advantageous.
SAT ANSWER EXPLANATIONS
n
READING AND WRITING: MODULE 1
10 SAT PRACTICE TEST #1 ANSWER EXPLANATIONS
QUESTION 16
Choice C is the best answer because it presents a finding that, if true, would
support the researchers’ hypothesis about the plants’ dependence on dissolving
rock. The text indicates that the roots of the two plant species grow directly into
quartzite rock, where hairs on the roots secrete acids that dissolve the rock.
The researchers hypothesize that the plants depend on this process because
dissolving rock opens spaces for the roots to grow and releases phosphates that
provide the plants with phosphorous, a vital nutrient. If the plants carry out this
process of dissolving rock even when the rock already has spaces into which
the roots could grow, that would support the researchers’ hypothesis because
it suggests that the plants are getting some advantage—such as access to
phosphorous—from the action of dissolving rock. If the plants don’t benefit from
dissolving rock, they would be expected to grow in the cracks that already exist,
as doing so would mean that the plants don’t have to spend energy creating and
secreting acids; if, however, the plants create new entry points by dissolving rock
even when cracks already exist, that would support the hypothesis that they
depend on dissolving rock for some benefit.
Choice A is incorrect because the existence of soil-inhabiting members of
the Velloziaceae family with similar root structures to those of the two species
discussed in the text wouldn’t support the researchers’ hypothesis that the species
discussed in the text depend on dissolving rock. If other such members exist, that
might suggest that the root structures can serve more functions than secreting
acids to dissolve rock (since dissolving rock may not be necessary for plants living
in soil), but that wouldn’t suggest anything about whether the species discussed
in the text benefit from dissolving rock. Choice B is incorrect because differences
in the proportions of citric and malic acid secreted by the two species would be
irrelevant to the hypothesis that the plants depend on dissolving rock. There’s no
information in the text to suggest that the proportion of each acid has any bearing
on the process of dissolving rock or on any benefits the plants might receive from
that process. Choice D is incorrect because if the two species thrive on rocks
without phosphates, that would weaken the researchers’ hypothesis that the plants
depend on dissolving rock partly because dissolving rock gives them access to
phosphates. If the plants can survive on rocks without getting a vital nutrient by
dissolving those rocks, then either the nutrient isn’t actually vital for those plants
or they can get the nutrient in some way other than by dissolving rocks.
QUESTION 17
Choice B is the best answer because it presents the conclusion that most
logically follows from the text’s discussion of the relationship between
atmospheric carbon dioxide and sauropod body size. The text establishes that
sauropods evolved to reach enormous sizes, and it notes that some scientists
have asserted that the cause of this phenomenon was increased plant production
that resulted from increased atmospheric carbon dioxide. The text goes on to
SAT ANSWER EXPLANATIONS
n
READING AND WRITING: MODULE 1
11 SAT PRACTICE TEST #1 ANSWER EXPLANATIONS
state, however, that atmospheric carbon dioxide levels didn’t increase around
the time of important periods in sauropods’ evolution of larger body sizes. If
significant periods of sauropod evolution toward larger sizes occurred without
increased atmospheric carbon dioxide levels, that suggests that the evolution of
larger sizes didn’t depend on increased carbon dioxide in the atmosphere.
Choice A is incorrect because the text doesn’t describe any fluctuations in
atmospheric carbon dioxide, so there’s no evidence in the text to support the
conclusion that such fluctuations had different effects on different sauropod
lineages. All that the text says about atmospheric carbon dioxide levels is that
there weren’t increases at particular points that correspond with key moments in
sauropod evolution. Choice C is incorrect because the text indicates that there
weren’t significant increases in atmospheric carbon dioxide around the time of
important periods in sauropods’ evolution toward larger body sizes, not that
atmospheric carbon dioxide was higher when the largest sauropods lived than
when sauropods first appeared. Choice D is incorrect because the text indicates
that atmospheric carbon dioxide levels didn’t increase at important periods in
sauropod evolution, not that higher levels would have affected that evolution.
The text provides no information about how higher levels of atmospheric carbon
dioxide might have affected sauropods.
QUESTION 18
Choice B is the best answer because it most logically completes the text’s
discussion of Anita Allen’s argument about judges citing philosophers in their
judicial opinions. The text indicates that judges sometimes cite philosophers
when writing their judicial opinions and that, according to Allen, judges tend to cite
philosophers whose views are in agreement with those of the judges themselves.
Allen claims, however, that the best judicial opinions consider potential objections
and rebut them, which suggests that judges may be able to strengthen their
opinions by including discussions of philosophers with views contrary to their own.
Choice A is incorrect because Allen’s claim is that judges could improve their
judicial opinions by citing philosophers who disagree with the views expressed
in the opinions, which would necessarily require judges to consult philosophical
works. Choice C is incorrect because there’s no discussion in the text about making
judicial opinions more easily understood by any particular group of readers. The
focus of the text is on Allen’s claim that judicial opinions could be strengthened
by the inclusion of discussions of philosophers whose views disagree with those
of the judges authoring the opinions. Choice D is incorrect because the text
presents Allen’s argument that discussing philosophers whose views judges
disagree with could strengthen judicial opinions, not that doing so could bring
those opinions into line with views that are popular among philosophers.
SAT ANSWER EXPLANATIONS
n
READING AND WRITING: MODULE 1
12 SAT PRACTICE TEST #1 ANSWER EXPLANATIONS
QUESTION 19
Choice A is the best answer. The convention being tested is pronoun-antecedent
agreement. The plural pronoun “they” agrees in number with the plural antecedent
customers.”
Choice B is incorrect because the singular pronoun “one” doesn’t agree in number
with the plural antecedent “customers.” Choice C is incorrect because the
second person pronoun “you” isn’t conventional as a substitute for “customers.”
It suggests that the audience (“you”) is the customer. Choice D is incorrect
because the singular pronoun “it” doesn’t agree in number with the plural
antecedent “customers.”
QUESTION 20
Choice A is the best answer. The convention being tested is punctuation use
between a main clause and a participial phrase. This choice correctly uses a
comma to mark the boundary between the main clause (“Epicurus…‘soul’”) and
the participial phrase (“positing…absence”) that provides additional information
about how Epicurus defined pleasure.
Choice B is incorrect because a colon can’t be used in this way to join a main
clause and a participial phrase. Choice C is incorrect because a semicolon can’t
be used in this way to join a main clause and a participial phrase. Choice D is
incorrect because it results in a rhetorically unacceptable sentence fragment
beginning with “positing.”
QUESTION 21
Choice C is the best answer. The convention being tested is the use of
possessive determiners. The plural possessive determiner “their” agrees in
number with the plural conjoined noun phrase “Watson and Crick” and thus
indicates that the findings were those of Watson and Crick.
Choice A is incorrect because “they’re” is the contraction for “they are,” not a
possessive determiner. Choice B is incorrect because “it’s” is the contraction
for “it is” or “it has,” not a possessive determiner. Choice D is incorrect because
the singular possessive determiner “its” doesn’t agree in number with the plural
conjoined noun phrase “Watson and Crick.”
QUESTION 22
Choice C is the best answer. The conventions being tested are punctuation use
between titles and proper nouns and between verbs and integrated quotations.
No punctuation is needed to set off the proper noun “Stina Chyn” from the title
that describes Chyn, “critic.” Because “Stina Chyn” is essential information
identifying the “critic,” no punctuation is necessary. Further, no punctuation
is needed between the verb “claims” and the following quotation because the
quotation is integrated into the structure of the sentence.
SAT ANSWER EXPLANATIONS
n
READING AND WRITING: MODULE 1
13 SAT PRACTICE TEST #1 ANSWER EXPLANATIONS
Choice A is incorrect because no punctuation is needed before or after the
proper noun “Stina Chyn.” Setting the critic’s name off with commas suggests
that it could be removed without affecting the coherence of the sentence,
which isn’t the case. Choice B is incorrect because no punctuation is needed
before or after the proper noun “Stina Chyn.” Setting the critic’s name off with
commas suggests that it could be removed without affecting the coherence
of the sentence, which isn’t the case. Additionally, no punctuation is needed
between “claims” and the integrated quotation. Choice D is incorrect because
no punctuation is needed between the verb “claims” and its subject, “critic Stina
Chyn.” Additionally, no punctuation is needed between the verb “claims” and the
integrated quotation.
QUESTION 23
Choice B is the best answer. The convention being tested is the use of finite and
nonfinite verb forms within a sentence. A main clause requires a finite verb to
perform the action of the subject (in this case, “some historians”), and this choice
supplies the finite present tense verb “claim” to indicate what some historians do.
Choice A is incorrect because the nonfinite participle “claiming” doesn’t supply
the main clause with a finite verb. Choice C is incorrect because the nonfinite
participle “having claimed” doesn’t supply the main clause with a finite verb.
Choice D is incorrect because the nonfinite to-infinitive “to claim” doesn’t supply
the main clause with a finite verb.
QUESTION 24
Choice A is the best answer. The convention being tested is colon use within
a sentence. A colon used in this way introduces information that illustrates or
explains information that has come before it. In this case, the colon introduces
the following explanation of why some roundworms in the Southern Hemisphere
move in the opposite direction of Earth’s magnetic field.
Choice B is incorrect because it results in a comma splice. A comma can’t be
used in this way to join two long independent clauses (“Researchers…food”
and “in…sources”) such as these. Choice C is incorrect because it results in a
run-on sentence. The two clauses (“Researchers…food” and “in…sources”) are
fused without punctuation. Furthermore, the conjunction “while” fails to indicate
that what follows is an explanation of why some roundworms in the Southern
Hemisphere move in the opposite direction of Earth’s magnetic field. Choice D is
incorrect because it results in a run-on sentence. The two clauses (“Researchers…
food” and “in…sources”) are fused without punctuation and/or a conjunction.
SAT ANSWER EXPLANATIONS
n
READING AND WRITING: MODULE 1
14 SAT PRACTICE TEST #1 ANSWER EXPLANATIONS
QUESTION 25
Choice B is the best answer. The convention being tested is pronoun-antecedent
agreement. The plural reflexive pronoun “themselves” agrees in number with the
plural antecedent “turtle barnacles,” correctly indicating what is attached to a sea
turtle shell.
Choice A is incorrect because the singular pronoun “it” doesn’t agree in number
with the plural antecedent “turtle barnacles.” Choice C is incorrect because it
results in an unclear and confusing sentence. In this context, it’s unclear what
the plural pronoun “them” refers to. Choice D is incorrect because the singular
reflexive pronoun “itself” doesn’t agree in number with the plural antecedent
“turtle barnacles.”
QUESTION 26
Choice A is the best answer. The convention being tested is subjectverb
agreement. The singular verb “allows” agrees in number with the singular subject
“landing.”
Choice B is incorrect because the plural verb “are allowing” doesn’t agree in
number with the singular subject “landing.” Choice C is incorrect because the
plural verb “have allowed” doesn’t agree in number with the singular subject
“landing.” Choice D is incorrect because the plural verb “allow” doesn’t agree in
number with the singular subject “landing.”
QUESTION 27
Choice A is the best answer. The convention being tested is the use of
punctuation to mark boundaries between supplements and clauses. The comma
after “equations” is used to separate the independent clause (“Hopper’s…
equation”) from the supplementary adverb phrase “though.” The colon after
“though” is used to mark the boundary between the clause ending with “though”
and the following clause (“as…age”). A colon used in this way introduces
information that illustrates or explains information that has come before it. In
this case, the colon after “though” introduces the following explanation of how
Hopper’s subsequent career would involve more than just solving equations: she
would become a pioneering computer programmer.
Choice B is incorrect because it results in a comma splice. A comma can’t be used
in this way to join two independent clauses (“Hopper’s…though” and “as…age”)
such as these. Choice C is incorrect because it results in an illogical sequence of
sentences. Placing the period after “equations” and beginning the next sentence
with “Though” illogically suggests that the following information (that Hopper
would help usher in the digital age) is contrary to the information in the previous
sentence (Hopper’s subsequent career would involve more than just solving
equations). Instead, the information that follows supports the information from
the previous sentence by explaining how her work and influence extended beyond
solely solving equations. Choice D is incorrect because it results in a run-on
sentence. The two independent clauses (“Hopper’s…though” and “as…age”) are
fused without punctuation.
SAT ANSWER EXPLANATIONS
n
READING AND WRITING: MODULE 1
15 SAT PRACTICE TEST #1 ANSWER EXPLANATIONS
QUESTION 28
Choice A is the best answer. The convention being tested is subject-modifier
placement. This choice ensures that the introductory phrase “upon recovering
two years later” appears immediately before the noun it modifies (“Henry”), clearly
establishing that Henry recovered two years later.
Choice B is incorrect because it results in a dangling modifier. The placement of
the noun phrase “the reign of Henry” immediately after the introductory phrase
illogically suggests that the reign of Henry recovered two years later. ChoiceC
is incorrect because it results in a dangling modifier. The placement of the noun
phrase “Henry’s reign” immediately after the introductory phrase illogically
suggests that Henry’s reign recovered two years later. Choice D is incorrect
because it results in a dangling modifier. The placement of the function word “it”
immediately after the introductory phrase illogically suggests that “it” recovered
two years later.
QUESTION 29
Choice D is the best answer. “For example” logically signals that the information
in this sentence—that The Emperor’s Babe is a novel conveyed in lines of
poetry—exemplifies the claim in the previous sentence about hybrid works that
incorporate elements of both novels and poems.
Choice A is incorrect because “by contrast” illogically signals that the information in
this sentence contrasts with the claim about hybrid works in the previous sentence.
Instead, the information demonstrates that Evaristo’s novel is an example of a
hybrid work. Choice B is incorrect because “consequently” illogically signals that
the information in this sentence is a consequence, or result, of the claim about
hybrid works in the previous sentence. Instead, the information demonstrates
that Evaristo’s novel is an example of a hybrid work. Choice C is incorrect because
“secondly” illogically signals that the information in this sentence is a second,
separate claim from the previous sentence’s claim about hybrid works. Instead, the
information demonstrates that Evaristo’s novel is an example of a hybrid work.
QUESTION 30
Choice C is the best answer. “By contrast” logically signals that the information
in this sentence—that dogs can see, hear, and smell by the end of two weeks—
contrasts with the preceding information (that wolves can smell but not see or
hear at the same age).
Choice A is incorrect because “in other words” illogically signals that the information
about domesticated dogs in this sentence paraphrases the information about
wolves in the previous sentence. Instead, the information about dogs contrasts with
what came before. Choice B is incorrect because “for instance” illogically signals
that the information about domesticated dogs in this sentence exemplifies the
information about wolves in the previous sentence. Instead, the information about
dogs contrasts with what came before. Choice D is incorrect because “accordingly”
illogically signals that the information about domesticated dogs in this sentence is
in accordance with, or results from, the information about wolves in the previous
sentence. Instead, the information about dogs contrasts with what came before.
SAT ANSWER EXPLANATIONS
n
READING AND WRITING: MODULE 1
16 SAT PRACTICE TEST #1 ANSWER EXPLANATIONS
QUESTION 31
Choice D is the best answer. “Increasingly” logically signals that the claim in
this sentence—that mathematicians are collaborating with their peers—marks
a change relative to what was traditionally done. As the previous sentence
explains, while mathematicians may have traditionally worked alone, evidence
points to a shift in the opposite direction. The claim describes the shift: a rise in
collaboration.
Choice A is incorrect because “similarly” illogically signals that the claim in
this sentence is similar to, but separate from, the previous claim about the
shift away from mathematicians working alone. Instead, the claim about the
rise in collaboration elaborates on the previous claim, describing the shift.
Choice B is incorrect because “for this reason” illogically signals that the claim
in this sentence is caused by the previous claim about the shift away from
mathematicians working alone. Instead, the claim about the rise in collaboration
elaborates on the previous claim, describing the shift. Choice C is incorrect
because “furthermore” illogically signals that the claim in this sentence is in
addition to the previous claim about the shift away from mathematicians working
alone. Instead, the claim about the rise in collaboration elaborates on the previous
claim, describing the shift.
QUESTION 32
Choice D is the best answer. The sentence presents both the study and its
findings, noting the study’s date and the researcher’s name as well as describing
what the researcher determined about the jawbones and how she determined it.
Choice A is incorrect. While the sentence describes the study and the
researcher’s initial assessment, it doesn’t present the study’s findings. Choice B is
incorrect. While the sentence describes the study and its focus, it doesn’t present
the study’s findings or the name of the researcher who conducted it. Choice C is
incorrect. While the sentence mentions the study’s methodology and provides
information about pterosaurs, it doesn’t present the study’s findings.
QUESTION 33
Choice C is the best answer. The sentence compares the two women’s
contributions to the march: Hedgeman worked behind the scenes to make sure a
woman speaker was included, whereas Bates actually spoke at the event.
Choice A is incorrect. While it acknowledges that the two women both
contributed to the march, it doesn’t indicate what Hedgeman did, so no
comparison is made. Choice B is incorrect. While the sentence provides
information about the two women, it doesn’t mention anything about Bates’s
contribution to the march. Choice D is incorrect. While the sentence indicates
that the two women both fought for civil rights, it doesn’t compare their individual
contributions to the march.
17 SAT PRACTICE TEST #1 ANSWER EXPLANATIONS
SAT ANSWER EXPLANATIONS
n
READING AND WRITING: MODULE 2
Reading and Writing
Module 2
(33 questions)
QUESTION 1
Choice B is the best answer because it most logically completes the text’s
discussion of Jacob Lawrence’s artistic process. In this context, “observant”
means watchful and perceptive. The text emphasizes that the “close attention
Lawrence paid to “all the details” of his neighborhood allowed him to reflect subtle
elements of “the beauty and vitality of the Black experience” in his artwork. This
context indicates that being observant of his surroundings was an important part
of Lawrence’s work as an artist.
Choice A is incorrect because the text gives no indication that Lawrence was
“skeptical,” or had an attitude of doubt in general or about particular things, let
alone that skepticism was important to him as an artist. Rather than indicating
that he was skeptical, the text focuses on how Lawrence paid careful attention
to everything around him and reflected his observations in his artwork. ChoiceC
is incorrect because the text gives no indication that Lawrence was “critical,”
which in this context would mean inclined to criticize harshly or unfairly. Rather
than indicating that Lawrence found fault in things, the text suggests that he
paid careful attention to everything around him and that his artwork reflects this
careful attention. Choice D is incorrect because the text doesn’t suggest that
Lawrence was “confident,” or self-assured. Rather than addressing how Lawrence
felt about himself and how that feeling affected his artistic process, the text
emphasizes the careful attention Lawrence paid to everything around him—
attention that allowed him to capture subtle elements of a particular place and
time in his artwork.
18 SAT PRACTICE TEST #1 ANSWER EXPLANATIONS
SAT ANSWER EXPLANATIONS
n
READING AND WRITING: MODULE 2
QUESTION 2
Choice D is the best answer because it most logically completes the text’s
discussion of the research that Lopes-Ferreira and her colleagues are conducting
on the stingray species Potamotrygon rex. As used in this context, “a substantial”
effect means an effect that is sizable or noteworthy. The text indicates that the
researchers are seeking to determine whether there are “considerable variations”
in the potency of stingray venom that are associated with variation in the
stingrays’ age and sex. This context suggests that the researchers want to find
out whether stingray age and sex have a substantial effect on venom toxicity.
Choice A is incorrect because there’s nothing in the text that suggests that the
researchers have been studying whether the stingrays’ age and sex have “a
disconcerting,” or an unsettling and disturbing, effect on the stingrays’ venom.
The text indicates that the researchers wish to determine if stingray age and
sex cause large variations in the toxicity of stingray venom, not if the effect of
age and sex is disconcerting. Choice B is incorrect because the text indicates
that researchers want to find out whether differences in stingray age and sex
produce differences in stingray venom, not that the researchers want to find out
whether age and sex have “an acceptable,” or a satisfactory, effect on venom.
The text makes no mention of what would make an effect on venom toxicity
acceptable and gives no indication that the researchers are interested in that
question. Choice C is incorrect because it wouldn’t make sense in context for the
researchers to be looking for “an imperceptible,” or an unnoticeable, effect of
age and sex on stingray venom. The text says that the researchers are trying to
determine if there are “considerable variations” in venom toxicity linked to age and
sex, not that the researchers are trying to find effects that they can’t perceive.
QUESTION 3
Choice A is the best answer because it most logically completes the text’s
discussion of diaphragm contractions and hiccups. In this context, “involuntarily”
means done without any control, or by reflex. The text explains that when a
person’s diaphragm repeatedly contracts and results in hiccups (which may be
beneficial for infants), those muscle contractions are “uncontrollable.” This context
indicates that the diaphragm contractions occur without the person’s control.
Choice B is incorrect because it wouldn’t support the logical relationship
established in the text’s discussion of diaphragm contractions and hiccups.
The text indicates that although specific causes for hiccups haven’t been
identified, it may be the case that the muscle contractions that occur have an
important purpose in infants. It wouldn’t make sense to say that even though
the contractions occur “beneficially,” or with a good or helpful effect, they might
play a positive role in infants’ breathing regulation. Choice C is incorrect because
the text indicates that the diaphragm contractions that result in hiccups are
“uncontrollable.” Because those muscle contractions are described as happening
automatically and without the person’s control, it wouldn’t make sense to
19 SAT PRACTICE TEST #1 ANSWER EXPLANATIONS
SAT ANSWER EXPLANATIONS
n
READING AND WRITING: MODULE 2
describe them as occurring “strenuously,” or in a way that requires great effort
or energy. Choice D is incorrect because the text doesn’t describe the quality of
the diaphragm contractions that result in hiccups beyond stating that they are
“uncontrollable.” Nothing in the text indicates that those muscle contractions
occur “smoothly,” or evenly and continuously.
QUESTION 4
Choice D is the best answer because it most logically completes the text’s
discussion about the relationship between fine art and fashion. As used in this
context, “intersect” means to connect or overlap. The text indicates that Jamie
Okuma challenges the position held by critics because her work can be seen
at an art museum and can be bought by the public from her online boutique.
The text also presents the critics’ view as being influenced by a perception that
fine artists create works that are “timeless” and meant for exhibition, whereas
fashion designers periodically produce new styles that are meant for purchase.
This context suggests that the critics believe that fine art and fashion tend not to
overlap—in other words, that they rarely intersect.
Choice A is incorrect because it wouldn’t make sense in context to say that
critics contend that fine art and fashion rarely “prevail,” or prove to be triumphant
or widespread. The text indicates that Okuma is an example of an artist who
demonstrates that it’s possible to make fine art that is also available to the public
as fashion. Choice B is incorrect because it wouldn’t make sense in context to
say that fine art and fashion rarely “succumb,” or surrender. The text establishes
that unlike what critics believe, Okuma creates works that are in art museums
and available for the public to purchase, suggesting that critics believe fine art
and fashion rarely overlap, not that they rarely succumb. Choice C is incorrect
because saying that critics believe that fine art and fashion rarely “diverge,” or
disagree or move in different directions, wouldn’t make sense in context. The text
presents Okuma’s work as both fine art and fashion, thereby undermining what
the critics assert. This suggests that the critics believe that fine art and fashion
rarely intersect rather than that the two rarely diverge.
QUESTION 5
Choice D is the best answer because it most logically completes the text’s
discussion of gender roles in Shakespeare’s comedies. As used in this context,
“prescribed” would mean laid down as rules. The text indicates that the characters
in the comedies often defy gender roles that are “socially dictated” (even if most
characters do return to those roles eventually) and that scholars have been
very interested in these acts of defiance. This context indicates that what the
characters are rebelling against are standards of behavior prescribed by the
society of the time.
20 SAT PRACTICE TEST #1 ANSWER EXPLANATIONS
SAT ANSWER EXPLANATIONS
n
READING AND WRITING: MODULE 2
Choice A is incorrect because saying that expectations about gender were
“interjected,” or suddenly inserted between other things, wouldn’t make sense
in context. There’s no suggestion in the text that the issue of gender roles was
inserted between other things or was an interruption in a larger discussion.
Choice B is incorrect because the text indicates that Shakespeare depicts
characters rebelling against expectations about gender that have been
“socially dictated,” not expectations that society has “committed,” or carried out,
entrusted, or promised. Choice C is incorrect because the text indicates that
Shakespeare depicts characters rebelling against expectations about gender
that have been “socially dictated,” not expectations that have been “illustrated,”
or clarified with examples. Although it’s possible for expectations about gender
roles to be illustrated, there’s nothing in the text to indicate that characters in
Shakespeare’s comedies rebel against illustrations of gender expectations.
QUESTION 6
Choice B is the best answer because it most logically completes the text’s
discussion of Ward and colleagues’ findings. As used in this context, “innocuous”
means mild or unharmful. The text describes the vibration and warming that Ward
and colleagues used to alleviate itching as “harmless applications” and goes on to
contrast these applications with another stimulus that actually offers more relief
even though it seems to be stronger and “less benign.” This context conveys the
idea that vibration and warming were innocuous stimuli.
Choice A is incorrect because the text focuses on a distinction between harmless
stimuli and those that seem to be less benign. Nothing in the text suggests
that any of the treatments are “deceptive,” or misleading; indeed, even the
less effective ones are described as offering some relief. Choice C is incorrect
because the text focuses on the amount of relief from itching offered by harmless
stimuli and those that seem to be less benign. The text doesn’t suggest that any
of these stimuli are “novel,” or original and new; heat, vibration, and electricity
aren’t new inventions. Choice D is incorrect because it wouldn’t make sense to
describe an application of vibration or warming as “impractical,” or not suitable
for use. The text indicates that these harmless applications are useful in that they
offer at least some temporary relief.
QUESTION 7
Choice D is the best answer because it most logically completes the text’s
discussion of the location of the province of Xoconochco within the Aztec Empire.
As used in this context, “peripheral” means situated toward the outer bounds
rather than the center. The text indicates that Xoconochco was located on a
coast, hundreds of kilometers away from the capital of the Aztec Empire. The
text also states that trade between the province and the capital required “a long
overland journey.” This context suggests that Xoconochco was situated toward
an edge of the empire’s territory rather than near its center.
21 SAT PRACTICE TEST #1 ANSWER EXPLANATIONS
SAT ANSWER EXPLANATIONS
n
READING AND WRITING: MODULE 2
Choice A is incorrect because it wouldn’t make sense in context to refer to
Xoconochco’s location within the Aztec Empire as “unobtrusive,” or not blatant
or undesirably prominent; it’s not clear how a province’s physical location would
or wouldn’t be blatant. Instead of focusing on how noticeable Xoconochco’s
location was, the text emphasizes the province’s distance from the capital of the
empire, pointing out that because of this distance trade between the two required
“a long overland journey.” Choice B is incorrect because the text indicates that
the province of Xoconochco was located on a coast far from the capital of the
Aztec Empire, not that it was “concealed,” or kept out of sight or hidden from
view. Nothing in the text suggests that Xoconochco was actually hidden such
that people couldn’t see it, and being hidden wouldn’t necessarily result in trade
between the province and the capital requiring “a long overland journey.” ChoiceC
is incorrect because to say that Xoconochco’s location within the Aztec Empire
was “approximate” would mean that the location either wasn’t precisely correct or
was close to some other location. Neither of these meanings would make sense
in context because the text indicates that Xoconochco’s location is known and
that it was far from the empire’s capital, so there’s no reason to characterize the
location as either not precisely correct or close to another location.
QUESTION 8
Choice A is the best answer because it most accurately describes the main
purpose of the text, which is to show that while Jane calmly goes about her daily
tasks, she is experiencing internal agitation about possibly seeking a new job.
At the start of the text, Jane says, “I went on with my day’s business tranquilly,”
indicating that she is outwardly calm. This outward calmness is then contrasted
with her intense internal restlessness, as Jane says that thoughts of leaving her job
keep running through her mind, that she is “involuntarily framing advertisements”
(meaning that she can’t stop herself from thinking up potential listings for jobs),
and that she often wonders what new “situations” (or jobs) would be like.
Choice B is incorrect because the text gives no indication of Jane’s feelings,
either positive or negative, about the people she works for at Thornfield Hall.
And rather than emphasizing that Jane feels particularly loyal to her employers,
the text focuses on her constant consideration of leaving her job. Choice C is
incorrect because the text gives no indication that Jane finds her current situation
fulfilling, or satisfying. Given that much of the text is focused on Jane’s thoughts
about possibly leaving her job for a new one, it might be the case that she finds
her situation challenging, but there is no evidence in the text that Jane also finds
that situation satisfying—she says nothing positive about her current job at all, in
fact. Choice D is incorrect because the text describes Jane as wondering about
getting a new job, not as determined to definitely do so. Jane keeps thinking
about reasons why she “should” quit her current job (indicating that she hasn’t yet
decided to) and imagining possible new situations she could find, but she says
at the end of the text that these thoughts “might germinate and bear fruit if they
could,” meaning that the thoughts haven’t yet led to a decision—that Jane isn’t
yet determined to get a new job somewhere else.
22 SAT PRACTICE TEST #1 ANSWER EXPLANATIONS
SAT ANSWER EXPLANATIONS
n
READING AND WRITING: MODULE 2
QUESTION 9
Choice B is the best answer because it reflects how the author of Text 2
would most likely respond to Text 1 based on the information provided. Text1
discusses the discovery of a regeneration-linked gene, EGR, in both three-
banded panther worms (which are capable of full regeneration) and humans (who
have relatively limited regeneration abilities). Text 1 characterizes this discovery
as “especially promising” and a sign of “exciting progress” in understanding
human regeneration. The author of Text 2, on the other hand, focuses on the
fact that the team that reported the EGR finding pointed out that while EGR’s
function in humans isn’t yet known, it’s likely very different from its function in
panther worms. Therefore, the author of Text 2 would most likely say that Text 1’s
enthusiasm about the EGR discovery is overly optimistic given Srivastava’s team’s
observations about EGR in humans.
Choice A is incorrect because the author of Text 2 explains that Srivastava and
her team explicitly reported that they haven’t yet identified how EGR functions
in humans; therefore, the author of Text 2 wouldn’t say that Text 1’s excitement
is reasonable for the stated reason. Instead, the author of Text 2 would likely
characterize Text 1’s excitement as premature and overly optimistic. ChoiceC is
incorrect because Text 1 does treat Srivastava’s team’s findings with enthusiasm;
it describes the discovery of EGR in both three-banded panther worms and
humans as promising and exciting. It would be illogical for the author of Text2
to say that because most others treat the discovery with enthusiasm, Text1’s
enthusiastic characterization of the discovery is unexpected. Choice D is
incorrect because Text 1 isn’t at all dismissive of Srivastava’s team’s findings;
instead, Text 1 is optimistic about the EGR discovery, characterizing it as
promising and exciting. There’s nothing in Text 2 to suggest that the author of
Text2 would say that Text 1’s praise for the discovery is dismissive, or disdainful.
QUESTION 10
Choice D is the best answer because it most accurately states the main idea of
the text. The speaker describes the experience of being “weary” and “tired” and
going to bed to seek “dear repose” (that is, sleep), but instead of sleeping, the
speaker is kept awake (“keep my drooping eyelids open wide”) by thoughts of a
friend (“my thoughts... [Begin] a zealous pilgrimage to thee”).
Choice A is incorrect because the text makes it clear that the speaker isn’t asleep;
thoughts about the friend are keeping the speaker awake (“keep my drooping
eyelids open wide”). Choice B is incorrect because the speaker isn’t talking about
taking a literal trip when referring to “a zealous pilgrimage.” Rather, the speaker is
referring to the experience of thinking about the friend, of taking “a journey in my
head.” Choice C is incorrect because the text indicates that the speaker and the
friend aren’t in the same place and having a conversation. Rather, the speaker is at
home and thinking of the friend, who is somewhere else (“from far where I abide”).
23 SAT PRACTICE TEST #1 ANSWER EXPLANATIONS
SAT ANSWER EXPLANATIONS
n
READING AND WRITING: MODULE 2
QUESTION 11
Choice D is the best answer because it presents a statement about how the Lord
Chancellor responds to the crowd that is supported by the text. The text indicates
that the people in the crowd are roaring and shouting “Bread!” or “Taxes!” and
presents them as not knowing what they really want. The Lord Chancellor’s
response is to ask what their shouting means but also to observe that they’re
shouting with “unanimity,” or total agreement. Clearly, this isn’t the case, which
supports the statement that the Lord Chancellor describes the crowd as being
united even though it’s not.
Choice A is incorrect because it isn’t supported by the text. Although the text
indicates that the Lord Chancellor asks about the meaning of the crowd’s
shouting, it doesn’t suggest that he knows what the crowd really wants. ChoiceB
is incorrect because the text doesn’t suggest that the Lord Chancellor wants
to speak to the crowd. Furthermore, the text doesn’t indicate that the crowd
wants to hear from the Sub-Warden. Although the crowd roars when asked
“Who roar for the Sub-Warden?” it’s unclear what the roaring means. Choice C is
incorrect because the text doesn’t suggest that the Lord Chancellor knows of or
sympathizes with the crowd’s demands. In addition, the text doesn’t indicate that
the crowd’s shouting annoys the Lord Chancellor, just that it causes him to keep
repeating “What can it all mean?”
QUESTION 12
Choice A is the best answer because it presents the quotation that most directly
illustrates the claim that Cather portrays Alexandra as having a deep emotional
connection to her natural surroundings. This quotation states that the country
meant a great deal to Alexandra and then goes on to detail several ways in which
her natural surroundings affect her emotionally: the insects sound like “the
sweetest music,” she feels as though “her heart were hiding” in the grass “with the
quail and the plover,” and near the ridges she feels “the future stirring.”
Choice B is incorrect because the quotation doesn’t suggest that Alexandra had
a deep emotional connection to her natural surroundings but instead describes
how she interacts with the people around her to learn more about crops, poultry,
and experiments with clover hay. Choice C is incorrect because the quotation
doesn’t suggest that Alexandra has a deep emotional connection to her natural
surroundings but instead describes her nighttime departure in a wagon. The
quotation says nothing about Alexandra’s emotional state. Choice D is incorrect
because the quotation doesn’t convey Alexandra’s deep emotional connection
to her natural surroundings; instead, this quotation describes how well she
understands the markets and livestock.
QUESTION 13
Choice B is the best answer because it provides the most direct support from the
table for the claim that two languages can convey similar amounts of information
even if they’re spoken at different rates. The table shows the approximate rates
at which five languages are spoken and the rates at which those five languages
convey information. Vietnamese is spoken at around 5.3 syllables per second,
24 SAT PRACTICE TEST #1 ANSWER EXPLANATIONS
SAT ANSWER EXPLANATIONS
n
READING AND WRITING: MODULE 2
whereas Spanish is spoken at around 7.7 syllables per second, but the two
languages convey information at very similar rates: Vietnamese at a rate of around
42.5 bits per second and Spanish at a rate of around 42.0 bits per second. Thus,
the description of Vietnamese conveying information at around the same rate
that Spanish does despite being spoken more slowly supports the claim in the
text that languages can convey the same amount of information even if spoken at
different rates.
Choice A is incorrect because it isn’t true that Thai and Hungarian have the lowest
rates of speech of the five languages shown. According to the table, Hungarian
is spoken at around 5.9 syllables per second, which is faster than Vietnamese
(5.3 syllables per second). Additionally, even if this statement were true, the
assertion that two languages are spoken the slowest and convey information the
slowest wouldn’t support the claim that languages can convey the same amount
of information even if they’re spoken at different rates. Choice C is incorrect
because it isn’t true that the fastest-spoken language (Spanish, at 7.7 syllables
per second) also conveys information the fastest: Spanish conveys information at
42.0 bits per second, which is slower than the 42.5 bits-per-second rate at which
Vietnamese conveys information. Additionally, even if this statement were true,
the assertion that the language spoken the fastest also conveys information the
fastest has no bearing on the claim that languages can convey the same amount
of information even if they’re spoken at different rates. Choice D is incorrect
because it isn’t true that Serbian conveys information faster than Spanish does.
According to the table, Serbian conveys information at a rate of around 39.1 bits
per second, which is slower than the 42.0 bits-per-second rate at which Spanish
conveys information.
QUESTION 14
Choice A is the best answer because it presents a finding that, if true, would
most strongly support the researchers’ claim that they found evidence that
experiencing awe can make people feel more connected to others and thus
more likely to behave altruistically (with beneficial and unselfish concern for
others). According to the text, the researchers tested for this effect by first having
participants look at either something known to be awe-inspiring (very tall trees)
or something ordinary (a plain building) and then purposely spilling pens near the
participants. The finding that participants who had looked at the trees helped pick
up significantly more pens than did participants who had looked at the building
would support the researchers’ claim by demonstrating that the people who had
experienced awe behaved more altruistically when the experimenter needed help
than the other participants did.
Choice B is incorrect because a finding about helpful participants using positive
words to describe the trees and the building after the experiment was over
wouldn’t have any bearing on the researchers’ claim that experiencing awe
increases altruistic behavior. The text doesn’t address the use of positive words to
describe things or suggest any connection between using such words and having
experienced awe, so that behavior wouldn’t serve as evidence that experiencing
awe played a role in promoting helpful behavior. Choice C is incorrect because a
finding that participants who didn’t help the experimenter were significantly more
25 SAT PRACTICE TEST #1 ANSWER EXPLANATIONS
SAT ANSWER EXPLANATIONS
n
READING AND WRITING: MODULE 2
likely than others to report having experienced awe whether they had looked at
the building or the trees would weaken the researchers’ claim that experiencing
awe increases altruistic behavior by suggesting that the opposite might be true—
that experiencing awe is in fact linked to choosing not to act in a way that benefits
someone else. Choice D is incorrect because a finding about participants noticing
that the experimenter had dropped the pens wouldn’t have any bearing on the
researchers’ claim about people behaving altruistically. Being aware of a challenge
or problem isn’t necessarily beneficial on its own and isn’t the same as offering
help, so the finding wouldn’t support the idea that experiencing awe increases
altruistic behavior.
QUESTION 15
Choice A is the best answer because it presents data from the table that most
effectively complete the statement about the rates at which employment shifted
in France and the United States. The text states that over the last two hundred
years employment in the agricultural sector has declined while employment in the
service sector has risen in both France and the US, and the data from the table
reflect these trends. The text asserts, however, that the transition from agriculture
to services “happened at very different rates in the two countries.” This assertion
is best supported by a comparison of data from 1900 and 1950: the table shows
that in those years, employment in agriculture went from 43% to 32% in France
(a decline of 11 percentage points) and from 41% to 14% in the US (a decline of
27 percentage points), and that employment in services went from 28% to 35%
in France (an increase of 7 percentage points) and from 31% to 53% in the US (an
increase of 22 percentage points). In other words, the rate of change was greater
in the US than in France for both sectors.
Choice B is incorrect because comparing the data for 1800 and 2012 would
suggest a similar rate of change in the two countries, not very different rates:
employment in agriculture went from 64% in 1800 to 3% in 2012 in France,
which is close to the change from 68% in 1800 to 2% in 2012 in the US, while
employment in services went from 14% in 1800 to 76% in 2012 in France, which
is close to the change from 13% in 1800 to 80% in 2012 in the US. Choice C is
incorrect because comparing the data for 1900 and 2012 would suggest a similar
rate of change in the two countries rather than very different rates: employment in
agriculture went from 43% in 1900 to 3% in 2012 in France, which is close to the
change from 41% in 1900 to 2% in 2012 in the US, while employment in services
went from 28% in 1900 to 76% in 2012 in France, which is close to the change
from 31% in 1900 to 80% in 2012 in the US. Choice D is incorrect because
comparing the data for 1800 and 1900 would suggest a similar rate of change in
the two countries, not very different rates: employment in agriculture went from
64% in 1800 to 43% in 1900 in France, which is fairly close to the change from
68% in 1800 to 41% in 1900 in the US, while employment in services went from
14% in 1800 to 28% in 1900 in France, which is close to the change from 13% in
1800 to 31% in 1900 in the US.
26 SAT PRACTICE TEST #1 ANSWER EXPLANATIONS
SAT ANSWER EXPLANATIONS
n
READING AND WRITING: MODULE 2
QUESTION 16
Choice C is the best answer because it presents a finding that, if true, would
support the researchers’ claim that archaeologists are unlikely to be replaced by
certain computer models. The text explains that although archaeologists hold
that categorizing pottery fragments relies on both objective criteria and instinct
developed through direct experience, researchers have found that a computer
model can categorize the fragments with the same degree of accuracy as the
humans can—a finding that has caused some archaeologists to worry that their
own work won’t be needed any longer. If survey results indicate that categorizing
pottery fragments limits the amount of time archaeologists can dedicate to other
important tasks that only human experts can do, that would mean that computer
models aren’t able to do all of the important things archaeologists do, thus
supporting the researchers’ claim that computer models are unlikely to replace
human archaeologists.
Choice A is incorrect because if it were true that the computer model could
categorize the pottery fragments much more quickly than the archaeologists
could, that would weaken the researchers’ claim that archaeologists are unlikely
to be replaced by certain computer models, since it would demonstrate that the
models could conduct the archaeologists’ work not only with equal accuracy
but also at a faster pace. Choice B is incorrect because the inability of both the
computer model and the archaeologists to accurately categorize all of the pottery
fragments presented wouldn’t support the researchers’ claim that archaeologists
are unlikely to be replaced by certain computer models. The text indicates that
some archaeologists are worried because the computer model’s accuracy
is equal to their own, and that could be the case whether both were perfectly
accurate or were unable to achieve complete accuracy. Choice D is incorrect
because survey results showing that few archaeologists received special training
in properly categorizing pottery fragments wouldn’t support the researchers’
claim that archaeologists are unlikely to be replaced by certain computer
models. The amount of special training in categorizing pottery fragments that
archaeologists have received has no direct bearing on whether computer
models’ success at categorizing fragments will lead to the models replacing the
archaeologists.
QUESTION 17
Choice A is the best answer because it presents the conclusion that most
logically follows from the text’s discussion of military veterans working in civilian
government jobs in the United States. The text indicates that the proportion of
military veterans working in civilian government jobs is considerably higher than
the proportion of military veterans in the population as a whole. The text also
notes that the unusually high representation of military veterans in these jobs
may be a result of the organizational structures shared by civilian government
entities and the military. Hence, it’s reasonable to infer that it’s the familiarity of
the structures of civilian government that makes jobs there particularly attractive
to military veterans.
27 SAT PRACTICE TEST #1 ANSWER EXPLANATIONS
SAT ANSWER EXPLANATIONS
n
READING AND WRITING: MODULE 2
Choice B is incorrect because the text doesn’t address what a typical relationship
between military service and later career preferences would be, and there’s no
indication that it’s atypical for veterans to work in civilian government jobs after
they’ve left the military. On the contrary, the text suggests that many military
veterans are drawn to such jobs. Choice C is incorrect because the text is
focused on the high representation of military veterans in civilian government
jobs and doesn’t address nonveterans or their possible interest in military
service. ChoiceD is incorrect because the text conveys that military veterans
may be particularly interested in civilian government jobs due to the familiarity
of organizational structures that are already in place, but there’s no reason to
think that this interest would mean that more civilian government jobs will start to
require military experience.
QUESTION 18
Choice D is the best answer because it most logically completes the text’s
discussion of Shultz’s finding about male tanagers. The text explains that because
carotenoids both contribute to deeply saturated feathers and offer health benefits,
having deeply saturated feathers is usually “an honest signal” (a true indication)
that a bird is generally fit. However, Shultz and others have found that certain
male tanagers can appear to have deeply saturated feathers even if they haven’t
consumed a diet rich in carotenoids, thanks to microstructures in their feathers
that manipulate light. If those birds aren’t necessarily eating carotenoid-rich diets,
they may actually be less fit than other birds that appear to have similarly saturated
feathers; this suggests that a male tanager’s appearance may function as a
dishonest signal, or a false indication, of the bird’s overall fitness.
Choice A is incorrect because Shultz’s finding suggests that some tanagers
can signal fitness without consuming the carotenoids that contribute to fitness,
thereby making those signals dishonest, not that tanagers can give honest
signals of their fitness without consuming carotenoids. Choice B is incorrect
because Shultz’s finding suggests that the microstructures in certain tanagers’
feathers can give a dishonest signal of fitness, not that the microstructures are
less effective than actual pigmentation for signaling fitness. Whether the signal
of fitness is honest or dishonest has no bearing on how effective the signal is:
a signal is effective if potential mates behave as though it’s true, regardless of
whether it’s actually true. Since there’s no information in the text about how
potential mates respond to the dishonest signals of some tanagers, there’s no
support for the idea that the dishonest signals are less effective than the honest
signals. Choice C is incorrect because Shultz’s finding suggests that certain male
tanagers may appear to be fitter than they actually are, not that scientists haven’t
determined why tanagers prefer mates with colorful appearances.
28 SAT PRACTICE TEST #1 ANSWER EXPLANATIONS
SAT ANSWER EXPLANATIONS
n
READING AND WRITING: MODULE 2
QUESTION 19
Choice C is the best answer. The convention being tested is subject-verb
agreement. The singular verb “has been” agrees in number with the singular
subject “writing.”
Choice A is incorrect because the plural verb “were” doesn’t agree in number with
the singular subject “writing.” Choice B is incorrect because the plural verb “have
been” doesn’t agree in number with the singular subject “writing.” Choice D is
incorrect because the plural verb “are” doesn’t agree in number with the singular
subject “writing.”
QUESTION 20
Choice B is the best answer. The convention being tested is the coordination
of main clauses within a sentence. This choice correctly uses a comma and the
coordinating conjunction “but” to join the first main clause (“the Alvarez…out”) and
the second main clause (“it left … extinctions”).
Choice A is incorrect because when coordinating two longer main clauses such
as these, it’s conventional to use a comma before the coordinating conjunction.
ChoiceC is incorrect because it results in a run-on sentence. The two main
clauses are fused without punctuation and/or a conjunction. Choice D is incorrect
because it results in a comma splice. Without a conjunction following it, a comma
can’t be used in this way to join two main clauses.
QUESTION 21
Choice C is the best answer. The convention being tested is the use of finite and
nonfinite verb forms within a sentence. The nonfinite present participle “forcing”
is correctly used to form a participial phrase that supplements the main clause
“those...cover,” describing the effects on monkeys of the lack of food sources.
Choice A is incorrect because the finite present tense verb “forces” can’t be used
in this way to supplement the main clause (“those...cover”). Choice B is incorrect.
While the nonfinite to-infinitive “to force” could be used to form a subordinate
clause that supplements the main clause (“those...cover”), to-infinitives
conventionally express purpose, and nothing in the sentence suggests that the
food sources become unavailable for the purpose of forcing monkeys to hunt
marine animals. Choice D is incorrect because the finite past tense verb “forced”
can’t be used in this way to supplement the main clause (“those...cover”).
29 SAT PRACTICE TEST #1 ANSWER EXPLANATIONS
SAT ANSWER EXPLANATIONS
n
READING AND WRITING: MODULE 2
QUESTION 22
Choice D is the best answer. The convention being tested is end-of-sentence
punctuation. This choice correctly uses a question mark to punctuate the
interrogative clause “could the blueberries thrive,” which asks a direct question at
the end of the sentence.
Choice A is incorrect because a period can’t be used in this way to punctuate
an interrogative clause, such as “could the blueberries thrive,” at the end of a
sentence. Choice B is incorrect because the context requires an interrogative
clause. The declarative clause “the blueberries could thrive” incorrectly indicates
that it was known that the blueberries could thrive in alkaline soil, whereas Michel
had yet to find this out. Choice C is incorrect because a question mark can’t be
used in this way to punctuate a declarative clause, such as “the blueberries could
thrive,” at the end of a sentence.
QUESTION 23
Choice D is the best answer. The convention being tested is the use of verbs to
express tense. In this choice, the present tense verb “is,” used in conjunction with
the word “today,” correctly indicates that Paik is currently considered the first
video artist.
Choice A is incorrect because the future-indicating verb “will be” doesn’t indicate
that Paik is currently considered the first video artist. Choice B is incorrect
because the past perfect tense verb “had been” doesn’t indicate that Paik is
currently considered the first video artist. Choice C is incorrect because the past
tense verb “was” doesn’t indicate that Paik is currently considered the first video
artist.
QUESTION 24
Choice C is the best answer. The convention being tested is punctuation use
between sentences. In this choice, the period is used correctly to mark the
boundary between the first sentence (“The...adjustments”) and the second
sentence (“Prior...days”). Because the adverbial phrase beginning with “prior”
indicates when changing a spreadsheet required redoing the sheet by hand, that
phrase belongs with the second sentence.
Choice A is incorrect because it results in a run-on sentence. Two sentences are
fused without punctuation and/or a conjunction. Choice B is incorrect because
it results in a comma splice. A comma can’t be used in this way to mark the
boundary between sentences. Choice D is incorrect. Without a comma preceding
it, the conjunction “and” can’t be used in this way to join the sentences.
30 SAT PRACTICE TEST #1 ANSWER EXPLANATIONS
SAT ANSWER EXPLANATIONS
n
READING AND WRITING: MODULE 2
QUESTION 25
Choice B is the best answer. The convention being tested is the use of finite and
nonfinite verb forms within a sentence. The modal “would,” which indicates the
future from a perspective in the past, should be accompanied by a nonfinite base
form verb. In this choice, the nonfinite base form verb “create” is used correctly in
conjunction with the nonfinite base form verb “increase” to describe what the lock
would do.
Choice A is incorrect because the finite present tense verb “creates” can’t be
used in this way with the modal “would” to describe what the lock would do.
Choice C is incorrect because the present participle “creating” can’t be used in
this way with the modal “would” to describe what the lock would do. Choice D is
incorrect because the finite past tense verb “created” can’t be used in this way
with the modal “would” to describe what the lock would do.
QUESTION 26
Choice D is the best answer. The convention being tested is subject-modifier
placement. This choice ensures that the modifying phrase “despite being cheap,
versatile, and easy to produce” appears immediately before the noun it modifies,
commercial plastics,” clearly establishing that the commercial plastics—and not
another noun in the sentence—are being described as cheap, versatile, and easy
to produce.
Choice A is incorrect because it results in a dangling modifier. The placement
of the function word “there” immediately after the modifying phrase illogically
and confusingly suggests that “there” is cheap, versatile, and easy to produce.
ChoiceB is incorrect because it results in a dangling modifier. The placement
of the noun “two problems” immediately after the modifying phrase illogically
suggests that the “problems” are cheap, versatile, and easy to produce. Choice C
is incorrect because it results in a dangling modifier. The placement of the noun
phrase “commercial plastics’ two associated problems” immediately after the
modifying phrase illogically suggests that the “problems” are cheap, versatile, and
easy to produce.
QUESTION 27
Choice C is the best answer. The convention being tested is the use of
punctuation between titles and proper nouns. No punctuation is needed to set
off the proper noun “Yuree Lee” from the title that describes Lee, “plant cell
biologist.” Because “Yuree Lee” is essential information identifying the “plant cell
biologist,” no punctuation is necessary.
Choice A is incorrect because no punctuation is needed. Choice B is incorrect
because no punctuation is needed. Choice D is incorrect because no punctuation
is needed before or after the proper noun “Yuree Lee.” Setting the biologist’s
name off with commas suggests that it could be removed without affecting the
coherence of the sentence, which isn’t the case.
31 SAT PRACTICE TEST #1 ANSWER EXPLANATIONS
SAT ANSWER EXPLANATIONS
n
READING AND WRITING: MODULE 2
QUESTION 28
Choice D is the best answer. “As a result” logically signals that the claim in this
sentence—that spiders can cling to and climb almost any surface—is because of
the previous information about the bonding properties of spiders’ spatulae.
Choice A is incorrect because “for instance” illogically signals that the claim in this
sentence exemplifies the information in the previous sentences. Instead, the claim
is because of the previous information about the bonding properties of spiders’
spatulae. Choice B is incorrect because “however” illogically signals that the claim
in this sentence contrasts with the information in the previous sentences. Instead,
the claim is because of the previous information about the bonding properties
of spiders’ spatulae. Choice C is incorrect because “similarly” illogically signals
that the claim in this sentence is similar to, but separate from, the information in
the previous sentences. Instead, the claim is because of the previous information
about the bonding properties of spiders’ spatulae.
QUESTION 29
Choice A is the best answer. “Still” logically signals that the information about
Sher-Gil in this sentence— that she longed to leave Paris and return to India—
contrasts with what one would expect after reading about Sher-Gil’s experiences
in Paris in the previous sentences.
Choice B is incorrect because “therefore” illogically signals that the information
about Sher-Gil in this sentence is a result or consequence of the descriptions
in the previous sentences. Instead, this information contrasts with what one
would expect after reading about Sher-Gil’s experiences in Paris. Choice C is
incorrect because “indeed” illogically signals that the information about Sher-
Gil in this sentence offers additional emphasis in support of the descriptions in
the previous sentences. Instead, this information contrasts with what one would
expect after reading about Sher-Gil’s experiences in Paris. Choice D is incorrect
because “furthermore” illogically signals that the information about Sher-Gil in
this sentence offers additional support for or confirmation of the descriptions in
the previous sentences. Instead, this information contrasts with what one would
expect after reading about Sher-Gil’s experiences in Paris.
QUESTION 30
Choice C is the best answer. “Similarly” logically signals that the activity
described in this sentence (Nancy Tuttle Craig distributing Votes for Women Tea
in her Los Angeles grocery stores) is like the activity described in the previous
sentence (the Woman’s Suffrage Party selling Equality Tea at fairs in San
Francisco). Together, the two examples support the preceding claim that “activists
across the state sold tea to promote the cause of suffrage.”
Choice A is incorrect because “for example” illogically signals that the activity
described in this sentence exemplifies the activity described in the previous
sentence. Instead, the two activities are similar, and both support the preceding
claim about selling tea to promote women’s right to vote. Choice B is incorrect
because “to conclude” illogically signals that the activity described in this
32 SAT PRACTICE TEST #1 ANSWER EXPLANATIONS
SAT ANSWER EXPLANATIONS
n
READING AND WRITING: MODULE 2
sentence concludes or summarizes the information in the previous sentences.
Instead, the activity is similar to the one described in the previous sentence, and
both support the preceding claim about selling tea to promote women’s right
to vote. Choice D is incorrect because “in other words” illogically signals that
the activity described in this sentence paraphrases the activity described in the
previous sentence. Instead, the two activities are similar, and both support the
preceding claim about selling tea to promote women’s right to vote.
QUESTION 31
Choice B is the best answer. The sentence compares the lengths of the two rail
tunnels, noting that the Channel Tunnel (about 31 miles long) is slightly shorter
than the Seikan Tunnel (roughly 33 miles long).
Choice A is incorrect. The sentence makes a generalization about the length of
some rail tunnels; it doesn’t compare the lengths of the two rail tunnels. ChoiceC
is incorrect. The sentence describes a single rail tunnel; it doesn’t compare the
lengths of the two rail tunnels. Choice D is incorrect. While the sentence mentions
the two rail tunnels, it doesn’t compare their lengths.
QUESTION 32
Choice B is the best answer because it provides both an explanation and an
example of “flauna.” The sentence explains that flauna, a combination of the
words “flora” and “fauna,” is a term used by Jon Ching to describe the plant-animal
hybrids in his paintings. The sentence also mentions an example of Ching’s flauna:
a parrot with leaves for feathers.
Choice A is incorrect. While the sentence partially explains what “flauna” is, it
doesn’t provide a full explanation or specific example of Ching’s flauna. Choice C
is incorrect. While the sentence partially explains what “flauna” is and includes a
title of a Ching painting, it doesn’t provide a full explanation or specific example of
Ching’s flauna. Choice D is incorrect. While the sentence partially explains what
“flauna” is and includes the titles of two Ching paintings, it doesn’t provide a full
explanation of Ching’s flauna.
QUESTION 33
Choice D is the best answer. By indicating that Taylor’s book was the only
Civil War memoir published by a Black woman, this sentence emphasizes the
uniqueness, or one-of-a-kind nature, of Taylor’s accomplishment.
Choice A is incorrect. While the sentence describes some of Taylor’s
accomplishments, it doesn’t emphasize the uniqueness of them. Choice B is
incorrect. While the sentence describes some of Taylor’s accomplishments,
it doesn’t emphasize that they were unique. Choice C is incorrect. While the
sentence provides information about Taylor’s book, it doesn’t emphasize what
made the book unique.
SAT ANSWER EXPLANATIONS
n
MATH: MODULE 1
33 SAT PRACTICE TEST #1 ANSWER EXPLANATIONS
Math
Module 1
(27 questions)
QUESTION 1
Choice B is correct.
of a quantity means
10
100
times the quantity. Therefore,
of
470
can be represented as
( )
10
100
470
, which is equivalent to
( )
0.10 470
,
or
47
. Therefore,
10%
of
470
is
47
.
Choice A is incorrect. This is
10%
of
370
, not
10%
of
470
. Choice C is
incorrect. This is
90%
of
470
, not
10%
of
470
. Choice D is incorrect. This is
470 10-
, not
10%
of
470
.
QUESTION 2
Choice C is correct. Subtracting
6
from both sides of the given equation yields
4 12x =
, which is the equation given in choice C. Since this equation is equivalent
to the given equation, it has the same solution as the given equation.
Choice A is incorrect and may result from conceptual or calculation errors.
ChoiceB is incorrect and may result from conceptual or calculation errors.
ChoiceD is incorrect and may result from conceptual or calculation errors.
QUESTION 3
Choice D is correct. The cost of the rental fee depends on the number of hours
the surfboard is rented. Multiplying
t
hours by
10
dollars per hour yields a rental
fee of
10t
dollars. The total cost of the rental consists of the rental fee plus the
25
dollar service fee, which yields a total cost of
25 10t+
dollars. Since the
person intends to spend a maximum of
75
dollars to rent the surfboard, the
total cost must be at most
75
dollars. Therefore, the inequality
25 10 75t
represents this situation.
Choice A is incorrect. This represents a situation where the rental fee, not the total
cost, is at most
75
dollars. Choice B is incorrect and may result from conceptual
SAT ANSWER EXPLANATIONS
n
MATH: MODULE 1
34 SAT PRACTICE TEST #1 ANSWER EXPLANATIONS
or calculation errors. Choice C is incorrect and may result from conceptual or
calculation errors.
QUESTION 4
Choice A is correct. It’s given that
( )
2
9gx x=+
. Substituting
25
for
( )
gx
in this
equation yields
2
25 9x=+
. Subtracting
9
from both sides of this equation yields
2
16 x=
. Taking the square root of each side of this equation yields
4x
. It
follows that
( )
25gx
= when the value of
x
is
4
or
4-
. Only
4
is listed among
the choices.
Choice B is incorrect and may result from conceptual or calculation errors.
ChoiceC is incorrect and may result from conceptual or calculation errors.
ChoiceD is incorrect and may result from conceptual or calculation errors.
QUESTION 5
Choice A is correct. The total number of possible outcomes for rolling a fair
14
-sided die is
14
. The number of possible outcomes for rolling a
2
is
1
. The
probability of rolling a
2
is the number of possible outcomes for rolling a
2
divided by the total number of possible outcomes, or
1
14
.
Choice B is incorrect. This is the probability of rolling a number no greater than
2
.
Choice C is incorrect. This is the probability of rolling a number greater than
2
.
Choice D is incorrect. This is the probability of rolling a number other than
2
.
QUESTION 6
The correct answer is
2,520
. There are
60
minutes in one hour. At a rate of
42
posters per minute, the number of posters produced in one hour can be
determined by
( )( )
42 posters 60 minutes
1 minute 1 hou r
, which is
2,520
posters per hour.
QUESTION 7
The correct answer is
30
. The value of
( )
fx
when
4x =
can be found
by substituting
4
for
x
in the given equation
( )
72fx x=+
. This yields
( ) ( )
4 74 2f =+
, or
( )
4 30f =
. Therefore, when
4x =
, the value of
( )
fx
is
30
.
QUESTION 8
Choice D is correct. Since
x
represents the number of
1
-point questions and
y
represents the number of
3
-point questions, the assignment is worth a total
of

13xy+
, or
3xy+
, points. Since the assignment is worth
70
points, the
equation
3 70xy+=
represents this situation.
Choice A is incorrect and may result from conceptual errors. Choice B is incorrect
and may result from conceptual errors. Choice C is incorrect and may result from
conceptual errors.
SAT ANSWER EXPLANATIONS
n
MATH: MODULE 1
35 SAT PRACTICE TEST #1 ANSWER EXPLANATIONS
QUESTION 9
Choice B is correct. It’s given that triangle
LMN
is similar to triangle
PQR
.
Corresponding angles of similar triangles are congruent. Since angle
M
and
angle
Q
correspond to each other, they must be congruent. Therefore, if the
measure of angle
M
is
o
53
, then the measure of angle
Q
is also
o
53
.
Choice A is incorrect and may result from concluding that angle
M
and angle
Q
are complementary rather than congruent. Choice C is incorrect and may result
from concluding that angle
M
and angle
Q
are supplementary rather than
congruent. Choice D is incorrect and may result from conceptual or calculation
errors.
QUESTION 10
Choice C is correct. The given system of linear equations can be solved by the
substitution method. Substituting
3x
-
for
y
from the first equation in the given
system into the second equation yields
( )
4 3 15xx+- =
, or
15x =
.
Choice A is incorrect and may result from conceptual or calculation errors.
ChoiceB is incorrect and may result from conceptual or calculation errors.
ChoiceD is incorrect. This is the absolute value of
y
, not the value of
x
.
QUESTION 11
Choice B is correct. The equation representing a linear model can be written in
the form
y a bx=+
, or
y bx a=+
, where
b
is the slope of the graph of the model
and
( )
0,a
is the y-intercept of the graph of the model. The scatterplot shows
that as the x-values of the data points increase, the y-values of the data points
decrease, which means the graph of an appropriate linear model has a negative
slope. Therefore,
0b
< . The scatterplot also shows that the data points are close
to the y-axis at a positive value of
y
. Therefore, the y-intercept of the graph of
an appropriate linear model has a positive y-coordinate, which means
0a >
. Of
the given choices, only choice B,
1.9 10.1yx=- +
, has a negative value for
b
, the
slope, and a positive value for
a
, the y-coordinate of the y-intercept.
Choice A is incorrect. The graph of this model has a y-intercept with a negative
y-coordinate, not a positive y-coordinate. Choice C is incorrect. The graph of this
model has a positive slope, not a negative slope, and a y-intercept with a negative
y-coordinate, not a positive y-coordinate. Choice D is incorrect. The graph of this
model has a positive slope, not a negative slope.
QUESTION 12
Choice C is correct. If a value of
x
satisfies
( )
0fx=
, the graph of
( )
y fx
= will
contain a point
( )
,0x
and thus touch the x-axis. Since there are
3
points at which
this graph touches the x-axis, there are
3
values of
x
for which
( )
0fx=
.
Choice A is incorrect and may result from conceptual or calculation errors.
ChoiceB is incorrect and may result from conceptual or calculation errors.
ChoiceD is incorrect and may result from conceptual or calculation errors.
SAT ANSWER EXPLANATIONS
n
MATH: MODULE 1
36 SAT PRACTICE TEST #1 ANSWER EXPLANATIONS
QUESTION 13
The correct answer is
41
. The number of cupcakes Vivian bought can be found
by first finding the amount Vivian spent on cupcakes. The amount Vivian spent
on cupcakes can be found by subtracting the amount Vivian spent on party hats
from the total amount Vivian spent. The amount Vivian spent on party hats can
be found by multiplying the cost per package of party hats by the number of
packages of party hats, which yields
$3 10
, or
$30
. Subtracting the amount
Vivian spent on party hats,
$30
, from the total amount Vivian spent,
$71
, yields
$71 $30-
, or
$41
. Since the amount Vivian spent on cupcakes was
$41
and each
cupcake cost
$1
, it follows that Vivian bought
41
cupcakes.
QUESTION 14
The correct answer is either
2
or
12-
. The left-hand side of the given equation
can be rewritten by factoring. The two values that multiply to
24-
and add
to
10
are
12
and
2-
. It follows that the given equation can be rewritten as
( )( )
12 2 0zz+ -=
. Setting each factor equal to
0
yields two equations:
12 0z +=
and
20z -=
. Subtracting
12
from both sides of the equation
12 0z +=
results
in
12z =-
. Adding
2
to both sides of the equation
20z -=
results in
2z =
. Note
that 2 and -12 are examples of ways to enter a correct answer.
QUESTION 15
Choice D is correct. Let
y
represent the number of cells per milliliter
x
hours
after the initial observation. Since the number of cells per milliliter doubles every
3
hours, the relationship between
x
and
y
can be represented by an
exponential equation of the form
( )
x
k
y ab=
, where
a
is the number of cells
per milliliter during the initial observation and the number of cells per milliliter
increases by a factor of
b
every
k
hours. It’s given that there were
300,000
cells
per milliliter during the initial observation. Therefore,
300,000a =
. It’s also given
that the number of cells per milliliter doubles, or increases by a factor of
2
, every
3
hours. Therefore,
2b =
and
3k =
. Substituting
300,000
for
a
,
2
for
b
, and
3
for
k
in the equation
( )
x
k
y ab=
yields
( )
3
300,000 2
x
y =
. The number
of cells per milliliter there will be
15
hours after the initial observation is the
value of
y
in this equation when
15x =
. Substituting
15
for
x
in the equation
( )
3
300,000 2
x
y =
yields
( )
15
3
300,000 2y
= , or
( )
5
300,000 2y
= . This is equivalent
to
( )
300,000 32y =
, or
9,600,000y =
. Therefore,
15
hours after the initial
observation, there will be
9,600,000
cells per milliliter.
Choice A is incorrect and may result from conceptual or calculation errors.
ChoiceB is incorrectand may result from conceptual or calculation errors.
ChoiceC is incorrect and may result from conceptual or calculation errors.
SAT ANSWER EXPLANATIONS
n
MATH: MODULE 1
37 SAT PRACTICE TEST #1 ANSWER EXPLANATIONS
QUESTION 16
Choice C is correct. Since each term of the given expression has a common
factor of
22
6xy
, it may be rewritten as
( )
( )
22 6 22
6 62xy x xy+
, or
( )
22 6
62xy x +
.
Choice A is incorrect. This expression is equivalent to
82
12xy
, not
82 22
6 12xy xy+
.
Choice B is incorrect. This expression is equivalent to
62
6xy
, not
82 22
6 12xy xy+
.
Choice D is incorrect. This expression is equivalent to
62 22
6 12xy xy+
, not
82 22
6 12xy xy+
.
QUESTION 17
Choice A is correct. It’s given that a neighborhood consists of a
2
-hectare
park and a
35
-hectare residential area and that the total number of trees in
the neighborhood is
3,934
. It’s also given that the equation
2 35 3,934xy+=
represents this situation. Since the total number of trees for a given area can
be determined by taking the number of hectares times the average number of
trees per hectare, this must mean that the terms
2x
and
35y
correspond to the
number of trees in the park and in the residential area, respectively. Since
2x
corresponds to the number of trees in the park, and
2
is the size of the park, in
hectares,
x
must represent the average number of trees per hectare in the park.
Choice B is incorrect and may result from conceptual errors. Choice C is incorrect
and may result from conceptual errors. Choice D is incorrect and may result from
conceptual errors.
QUESTION 18
Choice B is correct. The graph shown is a line passing through the points
( )
0,40
and
( )
60,0
. Since the relationship between
x
and
y
is linear, if two points
on the graph make a linear equation true, then the equation represents the
relationship.Substituting
0
for
x
and
40
for
y
in the equation in choiceB,
8 12 480xy+=
, yields
( ) ( )
8 0 12 40 480+=
, or
480 480=
, which is true.
Substituting
60
for
x
and
0
for
y
in the equation
8 12 480xy+=
yields
( ) ( )
8 60 12 0 480+=
, or
480 480=
, which is true. Therefore, the equation
8 12 480xy+=
represents the relationship between
x
and
y
.
Choice A is incorrect. The point
( )
0,40
is not on the graph of this equation, since
( )
40 8 0 12=+
, or
40 12=
, is not true. Choice C is incorrect. The point
( )
0,40
is not on the graph of this equation, since
( )
40 12 0 8=+
, or
40 8=
, is not true.
Choice D is incorrect. The point
( )
0,40
is not on the graph of this equation, since
( ) ( )
12 0 8 40 480+=
, or
320 480
= , is not true.
QUESTION 19
Choice D is correct. The area of a circle can be found by using the formula
π
2
Ar=
, where
A
is the area and
r
is the radius of the circle. It’s given that the
radius of circle A is
3n
. Substituting this value for
r
into the formula
π
2
Ar=
gives
( )
π
2
3An=
, or
π
2
9 n
. It’s also given that the radius of circle B is
129n
.
Substituting this value for
r
into the formula
π
2
Ar=
gives
( )
π
2
129An=
, or
SAT ANSWER EXPLANATIONS
n
MATH: MODULE 1
38 SAT PRACTICE TEST #1 ANSWER EXPLANATIONS
π
2
16,641 n
. Dividing the area of circle B by the area of circle A gives
π
π
2
2
16,641
9
n
n
,
which simplifies to
1,849
. Therefore, the area of circle B is
1,849
times the area of
circle A.
Choice A is incorrect. This is how many times greater the radius of circle B is than
the radius of circle A. Choice B is incorrect and may result from conceptual or
calculation errors. Choice C is incorrect. This is the coefficient on the term that
describes the radius of circle B.
QUESTION 20
The correct answer is
14
. The maximum value is the largest value in the data
set. The frequency refers to the number of times a data value occurs. The given
frequency table shows that for this data set, the data value
6
occurs three times,
the data value
7
occurs three times, the data value
8
occurs eight times, the
data value
9
occurs eight times, the data value
10
occurs nine times, the data
value
11
occurs eleven times, the data value
12
occurs nine times, the data value
13
occurs zero times, and the data value
14
occurs six times. Therefore, the
maximum data value in the data set is
14
.
QUESTION 21
The correct answer is
5
. The standard form of an equation of a circle in the
xy-plane is
( ) ( )
22
2
xh yk r- +- =
, where
h
,
k
, and
r
are constants, the
coordinates of the center of the circle are
( )
,hk
, and the length of the radius of
the circle is
r
. It′s given that an equation of the circle is
( ) ( )
22
2
29x yr- +- =
.
Therefore, the center of this circle is
( )
2,9
. It’s given that the endpoints of a
diameter of the circle are
( )
2,4
and
( )
2,14
. The length of the radius is the distance
from the center of the circle to an endpoint of a diameter of the circle, which can
be found using the distance formula,
( ) ( )
22
12 12
xx yy- +-
. Substituting the
center of the circle
( )
2,9
and one endpoint of the diameter
( )
2,4
in this formula
gives a distance of
( ) ( )
22
22 94- +-
, or
22
05+
, which is equivalent to
5
. Since
the distance from the center of the circle to an endpoint of a diameter is
5
, the
value of
r
is
5
.
QUESTION 22
Choice C is correct. It’s given that the measure of angle
R
is
π
2
3
radians, and
the measure of angle
T
is
π
5
12
radians greater than the measure of angle
R
.
Therefore, the measure of angle
T
is equal to
ππ
25
3 12
+
radians. Multiplying
π
2
3
by
4
4
to get a common denominator with
π
5
12
yields
π
8
12
. Therefore,
ππ
25
3 12
+
is
equivalent to
ππ
85
12 12
+
, or
π
13
12
. Therefore, the measure of angle
T
is
π
13
12
radians.
The measure of angle
T
, in degrees, can be found by multiplying its measure, in
SAT ANSWER EXPLANATIONS
n
MATH: MODULE 1
39 SAT PRACTICE TEST #1 ANSWER EXPLANATIONS
radians, by
π
180
. This yields
π
π
13 180
12
´
, which is equivalent to
195
degrees.
Therefore, the measure of angle
T
is
195
degrees.
Choice A is incorrect. This is the number of degrees that the measure of angle
T
is greater than the measure of angle
R
. Choice B is incorrect. This is the measure
of angle
R
, in degrees. Choice D is incorrect and may result from conceptual or
calculation errors.
QUESTION 23
Choice D is correct. Since the number of yards in
1
mile is
1,760
, the number of
square yards in
1
square mile is
( )( )
1,760 1,760 3,097,600=
. Therefore, if the area
of the town is
4.36
square miles, it is
( )
4.36 3,097,600 13,505,536
= , in square
yards.
Choice A is incorrect and may result from dividing the number of yards in a mile
by the square mileage of the town. Choice B is incorrect and may result from
multiplying the number of yards in a mile by the square mileage of the town.
Choice C is incorrect and may result from dividing the number of square yards in a
square mile by the square mileage of the town.
QUESTION 24
Choice D is correct. The equation of line
h
can be written in slope-intercept form
y mx b=+
, where
m
is the slope of the line and
( )
0,b
is the y-intercept of the
line. It’s given that line
h
contains the points
( )
18,130
,
( )
23,160
, and
( )
26,178
.
Therefore, its slope
m
can be found as
160 130
23 18
-
-
, or
6
. Substituting
6
for
m
in the
equation
y mx b=+
yields
6y xb=+
. Substituting
130
for
y
and
18
for
x
in this equation yields
( )
130 6 18 b=+
, or
130 108 b=+
. Subtracting
108
from
both sides of this equation yields
22 b=
. Substituting
22
for
b
in
6y xb=+
yields
6 22yx=+
. Since line
k
is the result of translating line
h
down
5
units, an
equation of line
k
is
6 22 5yx=+-
, or
6 17yx=+
. Substituting
0
for
y
in this
equation yields
0 6 17x=+
. Solving this equation for
x
yields
17
6
x =-
. Therefore,
the x-intercept of line
k
is
( )
17
6
,0-
.
Choice A is incorrect and may result from conceptual or calculation errors.
ChoiceB is incorrect and may result from conceptual or calculation errors.
ChoiceC is incorrect and may result from conceptual or calculation errors.
QUESTION 25
Choice C is correct. In the xy-plane, the graph of the line
yc=
is a horizontal
line that crosses the y-axis at
yc=
and the graph of the quadratic equation
2
9 100yx x=- + -
is a parabola. A parabola can intersect a horizontal line at
exactly one point only at its vertex. Therefore, the value of
c
should be equal to
the y-coordinate of the vertex of the graph of the given equation. For a quadratic
equation in vertex form,
( )
2
y ax h k= -+
, the vertex of its graph in the xy-plane is
( )
,hk
. The given quadratic equation,
2
9 100yx x=- + -
, can be rewritten as
SAT ANSWER EXPLANATIONS
n
MATH: MODULE 1
40 SAT PRACTICE TEST #1 ANSWER EXPLANATIONS
( ) ( ) ( )
22
2
999
222
2 100yx x
æö
÷
ç
=- - + + -
÷
ç
÷
÷
ç
èø
, or
( ) ( )
2
9 319
24
yx=- - + -
. Thus, the value of
c
is equal to
319
4
-
.
Choice A is incorrect and may result from conceptual or calculation errors.
ChoiceB is incorrect and may result from conceptual or calculation errors.
ChoiceD is incorrect and may result from conceptual or calculation errors.
QUESTION 26
Choice B is correct. The two given equations are equivalent because the second
equation can be obtained from the first equation by multiplying each side of the
equation by
5
. Thus, the graphs of the equations are coincident, so if a point
lies on the graph of one of the equations, it also lies on the graph of the other
equation. A point
( )
,xy
lies on the graph of an equation in the xy-plane if and only
if this point represents a solution to the equation. It is sufficient, therefore, to find
the point that represents a solution to the first given equation. Substituting the
x- and y-coordinates of choice B,
37
22
r
-+
and
r
, for
x
and
y
, respectively, in the
first equation yields
( )
37
22
2 37
r
r-++ =
, which is equivalent to
3 73 7rr- ++ =
, or
77=
. Therefore, the point
( )
37
22
,
r
r-+
represents a solution to the first equation
and thus lies on the graph of each equation in the xy-plane for the given system.
Choice A is incorrect and may result from conceptual or calculation errors.
ChoiceC is incorrect and may result from conceptual or calculation errors.
ChoiceD is incorrect and may result from conceptual or calculation errors.
QUESTION 27
The correct answer is
104
. An equilateral triangle is a triangle in which all three
sides have the same length and all three angles have a measure of
o
60
. The
height of the triangle,
3k
, is the length of the altitude from one vertex. The
altitude divides the equilateral triangle into two congruent 30-60-90right
triangles, where the altitude is the side across from the
o
60
angle in each30-60-
90right triangle. Since the altitude has a length of
3k
, it follows from the
properties of30-60-90right triangles that the side across from each
o
30
angle
has a length of
k
and each hypotenuse has a length of
2k
. In this case, the
hypotenuse of each30-60-90right triangle is a side of the equilateral triangle;
therefore, each side length of the equilateral triangle is
2k
. The perimeter of a
triangle is the sum of the lengths of each side. It’s given that the perimeter of the
equilateral triangle is
624
; therefore,
2 2 2 624kkk++=
, or
6 624k =
. Dividing
both sides of this equation by
6
yields
104k =
.
41 SAT PRACTICE TEST #1 ANSWER EXPLANATIONS
SAT ANSWER EXPLANATIONS
n
MATH: MODULE 2
Math
Module 2
(27 questions)
QUESTION 1
Choice A is correct. It’s given that Tilly earns
p
dollars for every
w
hours of
work. This can be represented by the proportion
p
w
. The amount of money,
x
, Tilly
earns for
39w
hours of work can be found by setting up the proportion
39
px
ww
=
.
This can be rewritten as
39pw xw=
. Dividing both sides by
w
results in
39xp=
.
Choice B is incorrect. This is the amount of money Tilly earns in dollars per hour,
not the amount of money Tilly earns for
39w
hours of work. Choice C is incorrect.
This is the amount of money Tilly earns for
w
hours of work plus
39
, not the
amount of money Tilly earns for
39w
hours of work. Choice D is incorrect. This is
the amount of money Tilly earns for
w
hours of work minus
39
, not the amount of
money Tilly earns for
39w
hours of work.
QUESTION 2
Choice D is correct. It′s given that Juan rides his bike at an average rate of
5.7
minutes per mile. The number of minutes it will take Juan to ride
x
miles can
be determined by multiplying his average rate by the number of miles,
x
, which
yields
5.7x
. Therefore, the function
( )
5.7mx x=
models the number of minutes it
will take Juan to ride
x
miles.
Choice A is incorrect and may result from conceptual errors.Choice B is incorrect
and may result from conceptual errors.Choice C is incorrect and may result from
conceptual errors.
QUESTION 3
Choice B is correct. Adding the second equation in the given system to the first
equation in the given system yields
( ) ( )
3 3 12 6x xy+- + = +-
, which is equivalent
to
6y =
.
42 SAT PRACTICE TEST #1 ANSWER EXPLANATIONS
SAT ANSWER EXPLANATIONS
n
MATH: MODULE 2
Choice A is incorrect and may result from conceptual or calculation errors.
ChoiceC is incorrect and may result from conceptual or calculation errors.
ChoiceD is incorrect and may result from conceptual or calculation errors.
QUESTION 4
Choice D is correct. In the given equation,
s
is the speed, in miles per hour, of a
certain car
t
seconds after it began to accelerate. Therefore, the speed of the
car, in miles per hour,
5
seconds after it began to accelerate can be found by
substituting
5
for
t
in the given equation, which yields
( )
40 3 5s =+
, or
55s =
.
Thus, the speed of the car
5
seconds after it began to accelerate is
55
miles per
hour.
Choice A is incorrect and may result from conceptual or calculation errors.
ChoiceB is incorrect and may result from conceptual or calculation errors.
ChoiceC is incorrect and may result from conceptual or calculation errors.
QUESTION 5
Choice D is correct. By the Pythagorean theorem, if a right triangle has a
hypotenuse with length
c
and legs with lengths
a
and
b
, then
222
cab=+
. In
the right triangle shown, the hypotenuse has length
c
and the legs have lengths
a
and
b
. It’s given that
4a =
and
5b =
. Substituting
4
for
a
and
5
for
b
in the
Pythagorean theorem yields
2 22
45c =+
. Taking the square root of both sides of
this equation yields
22
45c +
. Since the length of a side of a triangle must be
positive, the value of
c
is
22
45+
.
Choice A is incorrect and may result from conceptual or calculation errors.
ChoiceB is incorrect and may result from conceptual or calculation errors.
ChoiceC is incorrect and may result from conceptual or calculation errors.
QUESTION 6
The correct answer is
40
. Subtracting
5
from both sides of the given equation
yields
4 160x =
. Dividing both sides of this equation by
4
yields
40x =
.
Therefore, the solution to the given equation is
40
.
QUESTION 7
The correct answer is
7
. It’s given that the x-intercept of the graph shown is
( )
,0x
.
The graph passes through the point
( )
7,0
. Therefore, the value of
x
is
7
.
QUESTION 8
Choice B is correct. The y-intercept of the graph of a function in the xy-plane is
the point on the graph where
0x =
. It′s given that
( )
1
10
2fx x=-
. Substituting
0
for
x
in this equation yields
( )
( )
1
10
0 02f =-
, or
( )
02f =-
. Since it′s given that
( )
y fx=
, it follows that
2y =-
when
0x =
. Therefore, the y-intercept of the
graph of
( )
y fx=
in the xy-plane is
( )
0, 2-
.
43 SAT PRACTICE TEST #1 ANSWER EXPLANATIONS
SAT ANSWER EXPLANATIONS
n
MATH: MODULE 2
Choice A is incorrect and may result from conceptual or calculation errors.
ChoiceC is incorrect and may result from conceptual or calculation errors.
ChoiceD is incorrect and may result from conceptual or calculation errors.
QUESTION 9
Choice D is correct. If the graph of
( )
y gx=
is the result of shifting the graph
of
( )
y fx
= down
k
units in the xy-plane, the function
g
can be defined by an
equation of the form
( ) ( )
gx fx k=-
. It’s given that
( )
3
7fx x=
and the graph of
( )
y gx=
is the result of shifting the graph of
( )
y fx
= down
2
units. Substituting
3
7x
for
( )
fx
and
2
for
k
in the equation
( ) ( )
gx fx k=-
yields
( )
3
72gx x=-
.
Choice A is incorrect and may result from conceptual errors. Choice B is incorrect
and may result from conceptual errors. Choice C is incorrect. This equation
defines a function
g
for which the graph of
( )
y gx=
is the result of shifting the
graph of
( )
y fx
= up, not down,
2
units.
QUESTION 10
Choice A is correct. The solution to a system of equations is the ordered pair
( )
,xy
that satisfies all equations in the system. It’s given by the first equation in
the system that
7 10x +=
. Substituting
10
for
7x +
into the second equation
yields
2
10 y=
, or
100y =
. The x-coordinate of the solution to the system
of equations can be found by subtracting
7
from both sides of the equation
7 10x +=
, which yields
3x
=
. Therefore, the ordered pair
( )
3, 100
is a solution to
the given system of equations.
Choice B is incorrect and may result from conceptual or calculation errors.
ChoiceC is incorrect and may result from conceptual or calculation errors.
ChoiceD is incorrect and may result from conceptual or calculation errors.
QUESTION 11
Choice A is correct. Applying the distributive property, the given expression can
be written as
33
7 76 3x xx x+- +
. Grouping like terms in this expression yields
( )
( )
33
7 6 73x x xx- ++
. Combining like terms in this expression yields
3
10xx+
.
Choice B is incorrect and may result from conceptual or calculation errors.
ChoiceC is incorrect and may result from conceptual or calculation errors.
ChoiceD is incorrect and may result from conceptual or calculation errors.
QUESTION 12
Choice A is correct. It’s given that
( )
3
7pn n=
. Substituting
56
for
( )
pn
in this
equation yields
3
56 7n=
. Dividing each side of this equation by
7
yields
3
8 n=
.
Taking the cube root of each side of this equation yields
2 n=
. Therefore, when
( )
pn
is equal to
56
, the value of
n
is
2
.
Choice B is incorrect and may result from conceptual or calculation errors.
ChoiceC is incorrect and may result from conceptual or calculation errors.
ChoiceD is incorrect and may result from conceptual or calculation errors.
44 SAT PRACTICE TEST #1 ANSWER EXPLANATIONS
SAT ANSWER EXPLANATIONS
n
MATH: MODULE 2
QUESTION 13
The correct answer is
70
. Based on the figure, the angle with measure
o
110
and
the angle vertical to the angle with measure
o
x
are same side interior angles.
Since vertical angles are congruent, the angle vertical to the angle with measure
o
x
also has measure
o
x
. It’s given that lines
s
and
t
are parallel. Therefore, same
side interior angles between lines
s
and
t
are supplementary. It follows that
110 180x +=
. Subtracting
110
from both sides of this equation yields
70x =
.
QUESTION 14
The correct answer is
10
. The mean of a data set is calculated by dividing the sum
of the data values by the number of data values in the data set. For this data set,
the mean can be calculated as
6 8 16 4 17 26 8 5 5 5
10
++ ++ + ++++
, which is equivalent to
100
10
, or
10
.
QUESTION 15
Choice A is correct. For an exponential function of the form
( ) ( )
t
Et ab=
, where
a
and
b
are constants, the initial value of the function—that is, the value of the
function when
0t =
—is
a
and the value of the function increases by a factor
of
b
each time
t
increases by
1
. Since the function
( ) ( )
5 1.8
t
Et=
gives the
estimated number of employees at a restaurant and
t
is the number of years
since the restaurant opened, the best interpretation of the number
5
in this
context is the estimated number of employees when
0t =
, or when the restaurant
opened.
Choice B is incorrect and may result from conceptual errors. Choice C is incorrect
and may result from conceptual errors. Choice D is incorrect and may result from
conceptual errors.
QUESTION 16
Choice B is correct. For a quadratic function defined by an equation of the form
( ) ( )
2
gx ax h k= -+
, where
a
,
h
, and
k
are constants and
0a >
, the minimum
value of the function is
k
. In the given function,
1a =
,
0h =
, and
55k =
. Therefore,
the minimum value of the given function is
55
.
Choice A is incorrect. This is the value of
x
for which the given function reaches
its minimum value, not the minimum value of the function. Choice C is incorrect
and may result from conceptual or calculation errors. Choice D is incorrect and
may result from conceptual or calculation errors.
QUESTION 17
Choice C is correct. Because the value of the investment increases each
year, the function that best models how the value of the investment changes
over time is an increasing function. It′s given that each year, the value of the
investment increases by
0.49%
of its value the previous year. Since the value of
the investment changes by a fixed percentage each year, the function that best
45 SAT PRACTICE TEST #1 ANSWER EXPLANATIONS
SAT ANSWER EXPLANATIONS
n
MATH: MODULE 2
models how the value of the investment changes over time is an exponential
function. Therefore, the function that best models how the value of the
investment changes over time is an increasing exponential function.
Choice A is incorrect and may result from conceptual errors. Choice B is incorrect
and may result from conceptual errors. Choice D is incorrect and may result from
conceptual errors.
QUESTION 18
Choice C is correct. Let
x
be the 2015 population of Greenville. It’s given that
the population increased by
7%
from 2015 to 2016. The increase in population
can be written as
( )
0.07 x
. The 2016 population of Greenville is given as the sum
of the 2015 population of Greenville and the increase in population from 2015 to
2016. This can be rewritten as
( )
0.07xx+
, or
1.07x
. Therefore, the value of
k
is
1.07
.
Choice A is incorrect. This is the percent, represented as a decimal, that the
population increased from 2015 to 2016, not the value of
k
. Choice B is incorrect
and may result from conceptual or calculation errors. Choice D is incorrect. This is
the value of
k
if the population increased by
70%
, not
7%
, from 2015 to 2016.
QUESTION 19
Choice B is correct. Since
12
12
1=
, multiplying the exponent of the given expression
by
12
12
yields an equivalent expression:
11 12 132
12 12 144
aa
æöæö æ ö
÷÷ ÷
çç ç
÷÷ ÷
çç ç
÷÷ ÷
çç ç
÷÷ ÷
÷÷ ÷
çç ç
èøèø è ø
=
. Since
( )
132
1
144
144
132=
, the
expression
132
144
a
can be rewritten as
( )
1
132
144
a
æö
÷
ç
÷
ç
÷
ç
÷
÷
ç
èø
. Applying properties of exponents,
this expression can be rewritten as
( )
1
132
144
a
. An expression of the form
( )
1
k
m
,
where
0m >
and
0k >
, is equivalent to
k
m
. Therefore,
( )
1
132
144
a
is equivalent to
144
132
a
.
Choice A is incorrect and may result from conceptual or calculation errors.
ChoiceC is incorrect and may result from conceptual or calculation errors.
ChoiceD is incorrect and may result from conceptual or calculation errors.
QUESTION 20
The correct answer is
16
. The total cost of the party is found by adding the
onetime fee of the venue to the cost per attendee times the number of attendees.
Let
x
be the number of attendees. The expression
35 10.25x+
thus represents
the total cost of the party. It’s given that the budget is
$200
, so this situation
can be represented by the inequality
35 10.25 200x
. The greatest number of
attendees can be found by solving this inequality for
x
. Subtracting
35
from both
sides of this inequality gives
10.25 165x £
. Dividing both sides of this inequality
by
10.25
results in approximately
16.098x £
. Since the question is stated in
terms of attendees, rounding
x
down to the nearest whole number,
16
, gives the
greatest number of attendees possible.
46 SAT PRACTICE TEST #1 ANSWER EXPLANATIONS
SAT ANSWER EXPLANATIONS
n
MATH: MODULE 2
QUESTION 21
The correct answer is
28
. The given absolute value equation can be rewritten
as two linear equations:
4 4 112x -=
and
( )
4 4 112x- -=
, or
4 4 112x - =-
.
Adding
4
to both sides of the equation
4 4 112x -=
results in
4 116x =
. Dividing
both sides of this equation by
4
results in
29x =
. Adding
4
to both sides of the
equation
4 4 112x - =-
results in
4 108x =-
. Dividing both sides of this equation
by
4
results in
27x =-
. Therefore, the two values of
1x -
are
29 1-
, or
28
, and
27 1--
, or
28-
. Thus, the positive value of
1x -
is
28
.
Alternate approach: The given equation can be rewritten as
( )
4 1 112x -=
, which
is equivalent to
4 1 112x -=
. Dividing both sides of this equation by
4
yields
1 28x -=
. This equation can be rewritten as two linear equations:
1 28x -=
and
( )
1 28x- -=
, or
1 28x - =-
. Therefore, the positive value of
1x -
is
28
.
QUESTION 22
Choice A is correct. The volume of a cube can be found by using the formula
3
Vs=
, where
V
is the volume and
s
is the edge length of the cube. Therefore,
the volume of the given cube is
3
68V =
, or
314,432
cubic inches. The volume
of a sphere can be found by using the formula
π
3
4
3
Vr=
, where
V
is the volume
and
r
is the radius of the sphere. Therefore, the volume of the given sphere is
( )
π
3
4
3
34V =
, or approximately
164,636
cubic inches. The volume of the space
in the cube not taken up by the sphere is the difference between the volume of
the cube and volume of the sphere. Subtracting the approximate volume of the
sphere from the volume of the cube gives
314,432 164,636 149,796-=
cubic
inches.
Choice B is incorrect and may result from conceptual or calculation errors.
ChoiceC is incorrect and may result from conceptual or calculation errors.
ChoiceD is incorrect and may result from conceptual or calculation errors.
QUESTION 23
Choice B is correct. The standard form of an equation of a circle in the xy-plane
is
( ) ( )
22
2
,xh yk r- +- =
where the coordinates of the center of the circle are
( )
,hk
and the length of the radius of the circle is
r
. For the circle in the xy-plane
with equation
( ) ( )
22
5 3 16xy- +- =
, it follows that
2
16r =
. Taking the square root
of both sides of this equation yields
4r =
or
4r =-
. Because
r
represents the
length of the radius of the circle and this length must be positive,
4r =
. Therefore,
the radius of the circle is
4
. The diameter of a circle is twice the length of the
radius of the circle. Thus,
( )
24
yields
8
. Therefore, the diameter of the circle is
8
.
Choice A is incorrect. This is the radius of the circle.Choice C is incorrect. This is
the square of the radius of the circle.Choice D is incorrect and may result from
conceptual or calculation errors.
47 SAT PRACTICE TEST #1 ANSWER EXPLANATIONS
SAT ANSWER EXPLANATIONS
n
MATH: MODULE 2
QUESTION 24
Choice C is correct. For the form of the function in choice C,
( ) ( )
1
128 1.6
x
fx
-
=
,
the value of
( )
1f
can be found as
( )
11
128 1.6
-
, which is equivalent to
( )
0
128 1.6
, or
128
. Therefore,
128k =
, which is shown in
( ) ( )
1
128 1.6
x
fx
-
=
as the coefficient.
Choice A is incorrect and may result from conceptual or calculation errors.
ChoiceB is incorrect and may result from conceptual or calculation errors.
ChoiceD is incorrect and may result from conceptual or calculation errors.
QUESTION 25
Choice B is correct. Since the model estimates that the number of squirrels in
the population increased by a fixed percentage,
150%
, each year, the model can
be represented by an exponential equation of the form
( )
100
1
t
p
na=+
, where
a
is the estimated number of squirrels in the population at the end of
2015
, and
the model estimates that at the end of each year, the number is
%p
more than
the number at the end of the previous year. Since the model estimates that at the
end of each year, the number was
150%
more than the number at the end of the
previous year,
150p =
. Substituting
150
for
p
in the equation
( )
100
1
t
p
na=+
yields
( )
150
100
1
t
na=+
, which is equivalent to
( )
1 1.5
t
na=+
, or
( )
2.5
t
na=
. It’s given
that the estimated number of squirrels at the end of
2016
was
180
. This means
that when
1t =
,
180n =
. Substituting
1
for
t
and
180
for
n
in the equation
( )
2.5
t
na=
yields
( )
1
180 2.5a=
, or
180 2.5a=
. Dividing each side of this equation
by
2.5
yields
72 a=
. Substituting
72
for
a
in the equation
( )
2.5
t
na=
yields
( )
72 2.5
t
n =
.
Choice A is incorrect. This equation represents a model where at the end of each
year, the estimated number of squirrels was
150%
of, not
150%
more than, the
estimated number at the end of the previous year. Choice C is incorrect. This
equation represents a model where at the end of each year, the estimated number
of squirrels was
150%
of, not
150%
more than, the estimated number at the end
of the previous year, and the estimated number of squirrels at the end of
2015
,
not the end of
2016
, was
180
. Choice D is incorrect. This equation represents a
model where the estimated number of squirrels at the end of
2015
, not the end of
2016
, was
180
.
48 SAT PRACTICE TEST #1 ANSWER EXPLANATIONS
SAT ANSWER EXPLANATIONS
n
MATH: MODULE 2
QUESTION 26
Choice B is correct. Two lines are perpendicular if their slopes are negative
reciprocals, meaning that the slope of the first line is equal to
1-
divided by the
slope of the second line. Each equation in the given pair of equations can be
written in slope-intercept form,
y mx b=+
, where
m
is the slope of the graph of
the equation in the xy-plane and
( )
0,b
is the y-intercept. For the first equation,
571xy+=
, subtracting
5x
from both sides gives
7 51yx=- +
, and dividing both
sides of this equation by
7
gives
51
77
yx=- +
. Therefore, the slope of the graph
of this equation is
5
7
-
. For the second equation,
1ax by+=
, subtracting
ax
from
both sides gives
1by ax=- +
, and dividing both sides of this equation by
b
gives
1a
bb
yx=- +
. Therefore, the slope of the graph of this equation is
a
b
-
. Since the
graph of the given pair of equations is a pair of perpendicular lines, the slope of
the graph of the second equation,
a
b
-
, must be the negative reciprocal of the
slope of the graph of the first equation,
5
7
-
. The negative reciprocal of
5
7
-
is
1
5
7
-
æö
÷
ç
÷
-
ç
÷
ç
÷
ç
èø
, or
7
5
. Therefore,
7
5
a
b
-=
,or
7
5
a
b
=-
. Similarly, rewriting the equations
in choice B in slope-intercept form yields
10 1
77
yx=- +
and
1
22
a
bb
yx=- +
. It
follows that the slope of the graph of the first equation in choice B is
10
7
-
and the
slope of the graph of the second equation in choice B is
2
a
b
-
. Since
7
5
a
b
=-
,
2
a
b
-
is equal to
( )
( )
7
1
5
2
--
, or
7
10
. Since
7
10
is the negative reciprocal of
10
7
-
, the pair of
equations in choice B represents a pair of perpendicular lines.
Choice A is incorrect and may result from conceptual or calculation errors.
ChoiceC is incorrect and may result from conceptual or calculation errors.
ChoiceD is incorrect and may result from conceptual or calculation errors.
QUESTION 27
The correct answer is
289
. A quadratic equation of the form
2
0ax bx c+ +=
,
where
a
,
b
, and
c
are constants, has no real solutions when the value of the
discriminant,
2
4b ac-
, is less than
0
. In the given equation,
2
34 0x xc- +=
,
1a =
and
34b =-
. Therefore, the discriminant of the given equation can be expressed
as
( ) ( )( )
2
34 4 1 c--
, or
1,156 4c-
. It follows that the given equation has no real
solutions when
1,156 4 0c-<
. Adding
4c
to both sides of this inequality yields
1,156 4c<
. Dividing both sides of this inequality by
4
yields
289 c<
, or
289c >
.
It’s given that the equation
2
34 0x xc- +=
has no real solutions when
cn>
.
Therefore, the least possible value of
n
is
289
.
1
Scoring Your Paper
SAT Practice Test #1
IMPORTANT: This guide is for students who completed a paper version of the digital SAT practice test.
The scoring method used for this nondigital version isn’t exactly the same as the digital version.
Congratulations on completing an SAT
®
practice test.
To score your test, follow the instructions in this guide.
Scores Overview
Each assessment in the SAT Suite (SAT, PSAT/NMSQT®, PSAT
10, and PSAT
8/9) reports test
scores on a common scale.
For more details about scores, visit sat.org/scores.
The College Board Assessment Design & Development team developed the practice tests
using the same processes and review standards they used when developing the actual SAT.
1 Total Score
400–1600 Scale
Reading
and Writing
Modules 1 & 2 Modules 1 & 2
Math
Total Score
2 Section
Scores
200–800 Scale
2
How to Calculate Your Practice Test Scores
The worksheets on pages 4 and 5 help you calculate your test scores.
GET SET UP
1
In addition to your practice test, you’ll need the
conversion tables and answer key at the end of
this guide.
Have your students make time to take
the practice test.
It is one of the best ways to get ready
for the SAT.
The SAT
®
Practice Test for
Classroom Use
5
SAT Practice Test Worksheet:
Section and Total Scores
RAW SCORE
Reading and Writing
Section Score
Math
Section Score
(# OF CORRECT ANSWERS) LOWER UPPER LOWER UPPER
0 200 200 200 200
1 200 200 200 200
2 200 200 200 200
3 200 200 200 200
4 200 200 200 200
5 200 200 200 200
6 200 200 200 200
7 200 210 200 220
8 210 230 200 230
9 220 240 220 250
10 230 250 250 280
11 250 270 270 300
12 260 280 290 320
13 280 300 300 330
14 290 310 310 340
15 300 320 320 350
16 330 350 330 360
17 350 370 340 370
18 360 380 350 380
19 370 390 360 390
20 380 400 370 400
21 390 410 380 410
22 400 420 400 430
23 410 430 410 440
24 420 440 420 450
25 430 450 430 460
26 430 450 440 470
27 440 460 450 480
28 450 470 460 490
29 460 480 470 500
30 460 480 480 510
31 470 490 490 520
32 480 500 510 540
33 490 510 520 550
RAW SCORE
Reading and Writing
Section Score
Math
Section Score
(# OF CORRECT ANSWERS) LOWER UPPER LOWER UPPER
34 490 510 530 560
35 500 520 540 570
36 510 530 550 580
37 520 540 560 590
38 520 540 570 600
39 530 550 590 620
40 540 560 600 630
41 550 570 610 640
42 550 570 620 650
43 560 580 630 660
44 570 590 640 670
45 570 590 660 690
46 580 600 670 700
47 590 610 690 720
48 590 610 700 730
49 600 620 720 750
50 610 630 740 770
51 610 630 760 790
52 620 640 770 790
53 630 650 790 800
54 640 660 800 800
55 640 660
56 650 670
57 660 680
58 670 690
59 680 700
60 690 710
61 700 720
62 710 730
63 720 740
64 740 760
65 770 780
66 790 800
READING AND
WRITING SECTION
MODULE 1
RAW SCORE
(0-27)
READING AND
WRITING SECTION
MODULE 2
RAW SCORE
(0-27)
READING AND
WRITING
RAW SCORE
(0–54)
READING AND
WRITING
SECTION SCORE

MATH SECTION
MODULE 1
RAW SCORE
(0-22)
MATH SECTION
MODULE 2
RAW SCORE
(0-22)
MATH SECTION
RAW SCORE

TOTAL
SAT SCORE

=+
+ =
CONVERT
CONVERT
MATH
SECTION SCORE

+
READING AND WRITING
SECTION SCORE

=
Raw Score Conversion Table: Section Scores
Conversion:
Calculate Your Section and Total Scores
Use the table at the bottom of this page to convert your raw scores from
the previous worksheet into section and test scores.

SCORE YOUR PRACTICE TEST
2
Compare your answers to the answer key on
page 4, and count your total correct answers
for each section. Write the number of correct
answers for each section in the answer key
at the bottom of that section.
4
SAT Practice Test Worksheet:
Answer Key
Mark each of your correct answers below, then add them up to get your raw score on each module.
Math
Module 1 Module 2
Reading and Writing
Module 1 Module 2
MATH SECTION
RAW SCORE
(Total # of Correct Answers)
READING AND WRITING SECTION
RAW SCORE
(Total # of Correct Answers)
QUESTION #
CORRECT
MARK YOUR
CORRECT
ANSWERS
1 B
2 C
3 D
4 A
5 B
6 A
7 A
8 D
9 C
10 C
11 A
12 A
13 A
14 C
15 D
16 C
17 B
18 B
19 A
20 A
21 C
22 C
23 B
24 A
25 B
26 A
27 A
28 A
29 D
30 C
31 D
32 D
33 C
QUESTION #
CORRECT
MARK YOUR
CORRECT
ANSWERS
1 B
2 D
3 A
4 D
5 D
6 B
7 D
8 A
9 B
10 D
11 D
12 A
13 B
14 A
15 A
16 C
17 A
18 D
19 C
20 B
21 C
22 D
23 D
24 C
25 B
26 D
27 C
28 D
29 A
30 C
31 B
32 B
33 D
QUESTION #
CORRECT
MARK YOUR
CORRECT
ANSWERS
1 B
2 C
3 D
4 A
5 A
6 2520
7 30
8 D
9 B
10 C
11 B
12 C
13 41
14 2;-12
15 D
16 C
17 A
18 B
19 D
20 14
21 5
22 C
23 D
24 D
25 C
26 B
27 104
QUESTION #
CORRECT
MARK YOUR
CORRECT
ANSWERS
1 A
2 D
3 B
4 D
5 D
6 40
7 7
8 B
9 D
10 A
11 A
12 A
13 70
14 10
15 A
16 B
17 C
18 C
19 B
20 16
21 28
22 A
23 B
24 C
25 B
26 B
27 289
CALCULATE YOUR SCORES
3
Using your marked-up answer key and the
conversion tables, follow the directions on
page 5 to get your section and test scores.
5
SAT Practice Test Worksheet:
Section and Total Scores
RAW SCORE
Reading and Writing
Section Score
Math
Section Score
(# OF CORRECT ANSWERS) LOWER UPPER LOWER UPPER
0 200 200 200 200
1 200 200 200 200
2 200 200 200 200
3 200 200 200 200
4 200 200 200 200
5 200 200 200 200
6 200 200 200 200
7 200 210 200 220
8 210 230 200 230
9 220 240 220 250
10 230 250 250 280
11 250 270 270 300
12 260 280 290 320
13 280 300 300 330
14 290 310 310 340
15 300 320 320 350
16 330 350 330 360
17 350 370 340 370
18 360 380 350 380
19 370 390 360 390
20 380 400 370 400
21 390 410 380 410
22 400 420 400 430
23 410 430 410 440
24 420 440 420 450
25 430 450 430 460
26 430 450 440 470
27 440 460 450 480
28 450 470 460 490
29 460 480 470 500
30 460 480 480 510
31 470 490 490 520
32 480 500 510 540
33 490 510 520 550
RAW SCORE
Reading and Writing
Section Score
Math
Section Score
(# OF CORRECT ANSWERS) LOWER UPPER LOWER UPPER
34 490 510 530 560
35 500 520 540 570
36 510 530 550 580
37 520 540 560 590
38 520 540 570 600
39 530 550 590 620
40 540 560 600 630
41 550 570 610 640
42 550 570 620 650
43 560 580 630 660
44 570 590 640 670
45 570 590 660 690
46 580 600 670 700
47 590 610 690 720
48 590 610 700 730
49 600 620 720 750
50 610 630 740 770
51 610 630 760 790
52 620 640 770 790
53 630 650 790 800
54 640 660 800 800
55 640 660
56 650 670
57 660 680
58 670 690
59 680 700
60 690 710
61 700 720
62 710 730
63 720 740
64 740 760
65 770 780
66 790 800
READING AND
WRITING SECTION
MODULE 1
RAW SCORE
(0-27)
READING AND
WRITING SECTION
MODULE 2
RAW SCORE
(0-27)
READING AND
WRITING
RAW SCORE
(0–54)
READING AND
WRITING
SECTION SCORE

MATH SECTION
MODULE 1
RAW SCORE
(0-22)
MATH SECTION
MODULE 2
RAW SCORE
(0-22)
MATH SECTION
RAW SCORE

TOTAL
SAT SCORE

=+
+ =
CONVERT
CONVERT
MATH
SECTION SCORE

+
READING AND WRITING
SECTION SCORE

=
Raw Score Conversion Table: Section Scores
Conversion:
Calculate Your Section and Total Scores
Use the table at the bottom of this page to convert your raw scores from
the previous worksheet into section and test scores.

3
Get Section and
Total Scores
Your total score on an SAT practice test is the sum of your scores in the
modules for both the Reading and Writing and Math sections. To get your
total score, you’ll convert what we call the “raw score” for each section—
the number of questions you got right in that section—into the “scaled
score” for that section, and then calculate the total score.
GET YOUR READING AND WRITING SECTION SCORE
Calculate your SAT Reading and Writing section score
(it’s on a scale of 200–800).
1
Use the answer key on page 4 to count the number of correct
answers you got on module 1 and module 2.
2
To determine your Reading and Writing raw score, add the number
of correct answers you got on module 1 and module 2.
3
Use the Raw Score Conversion Table: Section Scores on page 5
to turn your raw score into your Reading and Writing section score.
GET YOUR MATH SECTION SCORE
Calculate your SAT Math section score (it’s on a scale of 200–800).
1
Use the answer key on page 4 to count the number of correct
answers you got on module 1 and module 2.
2
To determine your Math raw score, add the number of correct
answers you got on module 1 and module 2.
3
Use the Raw Score Conversion Table: Section Scores on page 5
to turn your raw score into your Math section score.
GET YOUR TOTAL SCORE
Add your Reading and Writing section score to your Math section score.
The result is your total score on the SAT practice test, on a scale of 400–1600.
1 Total Score
400–1600 Scale
2 Section
Scores
200–800 Scale
Reading
and Writing
Modules 1 & 2 Modules 1 & 2
Math
Total Score
Your total score on the SAT practice test
is the sum of your Reading and Writing
section score and your Math section score.
Use the worksheets on
pages 4 and 5 to calculate your
section and total scores.
4
SAT Practice Test Worksheet:
Answer Key
Mark each of your correct answers below, then add them up to get your raw score on each module.
Math
Module 1 Module 2
Reading and Writing
Module 1 Module 2
QUESTION #
CORRECT
MARK YOUR
CORRECT
ANSWERS
1 B
2 C
3 D
4 A
5 B
6 A
7 A
8 D
9 C
10 C
11 A
12 A
13 A
14 C
15 D
16 C
17 B
18 B
19 A
20 A
21 C
22 C
23 B
24 A
25 B
26 A
27 A
28 A
29 D
30 C
31 D
32 D
33 C
QUESTION #
CORRECT
MARK YOUR
CORRECT
ANSWERS
1 B
2 D
3 A
4 D
5 D
6 B
7 D
8 A
9 B
10 D
11 D
12 A
13 B
14 A
15 A
16 C
17 A
18 D
19 C
20 B
21 C
22 D
23 D
24 C
25 B
26 D
27 C
28 D
29 A
30 C
31 B
32 B
33 D
READING AND WRITING SECTION
RAW SCORE
(Total # of Correct Answers)
QUESTION #
CORRECT
MARK YOUR
CORRECT
ANSWERS
1 B
2 C
3 D
4 A
5 A
6 2520
7 30
8 D
9 B
10 C
11 B
12 C
13 41
14 2; -12
15 D
16 C
17 A
18 B
19 D
20 14
21 5
22 C
23 D
24 D
25 C
26 B
27 104
QUESTION #
CORRECT
MARK YOUR
CORRECT
ANSWERS
1 A
2 D
3 B
4 D
5 D
6 40
7 7
8 B
9 D
10 A
11 A
12 A
13 70
14 10
15 A
16 B
17 C
18 C
19 B
20 16
21 28
22 A
23 B
24 C
25 B
26 B
27 289
MATH SECTION
RAW SCORE
(Total # of Correct Answers)
5
SAT Practice Test Worksheet:
Section and Total Scores
Conversion:
Calculate Your Section and Total Scores
Use the table at the bottom of this page to convert your raw scores from
the previous worksheet into section and test scores.
READING AND
WRITING SECTION
MODULE 1
RAW SCORE
(0–27)
+
READING AND
WRITING SECTION
MODULE 2
RAW SCORE
(0–27)
=
READING AND
WRITING
RAW SCORE
(0–54)
CONVERT
READING AND
WRITING
SECTION SCORE
(200–800)
MATH SECTION
MODULE 1
RAW SCORE
(0–22)
+
MATH SECTION
MODULE 2
RAW SCORE
(0–22)
=
MATH SECTION
RAW SCORE
(0–44)
CONVERT
MATH
SECTION SCORE
(200–800)
+
READING AND WRITING
SECTION SCORE
(200–800)
=
TOTAL
SAT SCORE
(400–1600)
Raw Score Conversion Table: Section Scores
RAW SCORE
Reading and Writing
Section Score
Math
Section Score
(# OF CORRECT ANSWERS) LOWER UPPER LOWER UPPER
0 200 200 200 200
1 200 200 200 200
2 200 200 200 200
3 200 200 200 200
4 200 200 200 200
5 200 200 200 200
6 200 200 200 200
7 200 210 200 220
8 210 230 200 230
9 220 240 220 250
10 230 250 250 280
11 250 270 270 300
12 260 280 290 320
13 280 300 300 330
14 290 310 310 340
15 300 320 320 350
16 330 350 330 360
17 350 370 340 370
18 360 380 350 380
19 370 390 360 390
20 380 400 370 400
21 390 410 380 410
22 400 420 400 430
23 410 430 410 440
24 420
440 420 450
25 430 450 430 460
26 430 450 440 470
27 440 460 450 480
28 450 470 460 490
29 460 480 470 500
30 460 480 480 510
31 470 490 490 520
32 480 500 510 540
33 490 510 520 550
RAW SCORE
R
eading and Writing
S
ection Score
Math
Section Score
(# OF CORRECT ANSWERS) LOWER UPPER LOWER UPPER
34 490 510 530 560
35 500 520 540 570
36 510 530 550 580
37 520 540 560 590
38 520 540 570 600
39 530 550 590 620
40 540 560 600 630
41 550 570 610 640
42 550 570 620 650
43 560 580 630 660
44 570 590 640 670
45 570 590 660 690
46 580 600 670 700
47 590 610 690 720
48 590 610 700 730
49 600 620 720 750
50 610 630 740 770
51 610 630 760 790
52 620 640 770 790
53 630 650 790 800
54 640 660 800 800
55 640 660
56 650 670
57 660 680
58 670 690
59 680 700
60 690
710
61 700 720
62 710 730
63 720 740
64 740
760
65 770 780
66 790 800
© 2022 College Board.   2223-BB-622